Sunteți pe pagina 1din 165
mu PROBLEMS mmm 144 Problems of the Austrian-Polish Mathematics Competition 1978 - 1993 Compiled and with solutions by MARCIN E. KUCZMA University of Warsaw Published by... The Academic Distribution Center PROBLEMS 144 PROBLEMS of the AUST RIAN-POLISH MATHEMATICS COMPETITION (1978-1993) Compiled and with solutions by MARCIN E. KUCZMA University of Warsaw FOREWORD by WALTER E. MIENTKA University of Nebraska at Lincoln The ACADEMIC DISTRIBUTION CENTER Freeland, Maryland Published by The Academic Distribution Center 1216 Walker Rd. Freeland, MD 21053 USA Manuscript received November, 1993 and revised May, 1994 Text typeset by Marcin B. Kuczma Diagrams typeset by Rafal Sztencel Solutions Copyright © 1994 by Marcin Emil Kuczma. All rights reserved. No part of the Solutions in this book may be reproduced or transmitted in any form or by any means, electronic or mechanical, including photocopying, recording, or by any information storage and retrieval system, without the written permission of the Academic Distribution Center, except where permitted by law. Library of Congress Catalog Card Number: 94-71511 ISBN 0-9640959-0-4 Printed in the United States of America FOREWORD There are many paths to the acquisition of mathematical knowledge and applying it to the achievement of new knowledge. There is no magic wand which will guarantee success to all who have the will, desire and time to reach that goal. ‘There is one indisputable common thread which is woven throughout the development of students of mathematics, and that is the challenge of solv- ing problems. The history of mathematics has recorded the contributions by many great mathematicians who have solved or proposed challenging problems. This has led to results which in many cases were the catalyst to the formation of new branches of Mathematics. One must include the contributions of Archimedes, Descartes, Fermat, Newton, Euler, Gauss and Hilbert, among those recognized for their achievements. In this manuscript, Dr. Marcin E. Kuczma’s solutions to the problems posed at the Austrian-Polish Mathematics Competitions are presented with elegant style and clarity, reflecting the long standing tradition associated with Polish mathematicians. He is identified throughout the world of math- ematics as a research mathematician, creator of intriguing problems and as one who possesses unusual mathematical ability. In fact, he has con- tributed to the problems sections of The American Mathematical Monthly, Crux Mathematicorum, Journal of Recreational Mathematics, Elemente der Mathematik and Delta. Dr. Kuczma has received many honors and awards for his research and teaching, and for his work associated with the Polish and Interna- tional Mathematical Olympiad (IMO). Especially significant is the fact that among the proposed problems he submitted for the IMO, two were selected by the Jury for the competition. A testimony of the significance of his work was recently acknowledged by the World Federation of National Mathematics Competitions which awarded him the prestigious David Hilbert International Award. This award was established “to recognize contributions of mathematicians which have played a significant role in the development of mathematical challenges at the international level and which have been a stimulus for the enrichment of mathematics learning”. It is a privilege for me to have this opportunity to introduce you to Dr. Kuczma and this elegant set of problems and solutions. Walter E. Mientka Executive Director American Mathematics Competitions Professor of Mathematics University of Nebraska at Lincoln U.S.A. iii ACKNOWLEDGEMENTS ‘There are many interesting problems in this book. They have been devised by numerous mathematicians from both countries competing in the APMC. The habit is that the problem proposals are supplied just by Austria or Poland; the actual proposers are anonymous... Let them re- main so. Although, in many cases, I know exactly who the proposer was, there are far more cases where I do not know that, and it would be unfair to give the names in some instances and not to give them in the others. But it must be remembered that those mathematicians are, in fact, the true Authors of this book. My hat is off to them arid my gratitude is most sincere. I have learned a lot by studying those problems and I have had a good deal of true pleasure; it was a fascinating adventure forme. > ‘As pointed out in The APMC section, the solutions presented here have had several sources of inspiration. I am grateful to Professor Andrzej Schinzel (Polish Academy of Sciences) for his kind permission to reproduce his solution to one of the problems. The problems of the early competitions, with solution sketches, had been reported in the yearly Polish Mathemati- cal Olympiad brochures by Dr. Maciej Bryriski, a colleague of mine; I have taken over several of his ideas. ‘The manuscript has been reviewed by Professors Titu Andreescu (Illi- nois Mathematics and Science Academy) and Svetoslav Savchev (the editor of Matematika, Sofia, Bulgaria). Many improvements in the text are owed to their friendly criticism. If the result is not perfect, it is only because I was not always obedient enough in following their suggestions. Professor Walter E. Mientka (University of Nebraska, Lincoln), the vice- President of the World Federation of National Mathematics Competitions, is the man who encouraged me to write this compilation and whose warm words of encouragement assisted me all the time during the preparation of this book; he equipped it with a Foreword, full of words of enthusiasm, undeserved by the author. His wife Gretel and his daughter Rebecca under- took the task of proofreading the manuscript and correcting the language. That this book appears, is due to the efforts of Rebecca A. Whitaker, the Director of the Academic Distribution Center. Her patience in an end- less exchange of fax letters, always optimistic and polite, her readiness to respect the author’s most capricious wishes, and her advice on dozens of editorial details, were to me of value that cannot be over-appreciated To all those persons I am directing my warmest thanks for their help, their efforts, their never-ceasing cordiality! ‘The last word of thanks goes to my wife, Malgosia. The final weeks of my work on getting the book ready for print were, without doubt, not the easiest ones in her life; and yet she was patient, understanding, smiling. ‘Thank you, dear Marcin E. Kuczma CONTENTS FOREWORD ACKNOWLEDGEMENTS The APMC - To the Reader PROBLEMS SOLUTIONS iii a vi The APMC To the Reader ‘We are presenting you with a collection of 144 rather challenging problems in high school “olympiad-style” mathematics. The problems come from the Austrian-Polish Mathematics Competition (the APMC). A few words of information about the APMC are in order. ‘The Competition arose from a bilateral agreement on cultural exchange between the Polish and the Austrian Ministers of Education. The event takes place annually in July, or late June, in Austria or in Poland (alter- nately), the host country being Poland in even-numbered years and Austria in odd-numbered years. APMC-1 was organized in 1978 in Poland; accord- ingly, APMC-16 took place in 1993 in Austria. ‘The contestants are high-school students (or those who just left school), not participating in the International Mathematical Olympiad (IMO) the same year. Normally, the top six students at the national math olympiad of each country go to the IMO and the next-to-top six form the APMC team. ‘The Competition consists of two parts, the individual contest and the team contest. The style and format of the Austrian and the Polish Mathe- matical Olympiads (their final rounds) are very similar to those of the IMO; so the APMC individual contest follows the same pattern: a two day exam, 3 problems to be solved in 4} hours (each day). On the third day, the team contest takes place, with 3 problems to be solved in 4 hours. The teams occupy two separate rooms; the students of each team work jointly and organize their work in an unrestricted manner, without being supervised. Problems of the individual contest are numbered 1 through 6, while those of the team contest are numbered 7 through 9. The three days of contests are then followed by a recreational program of sight-seeing and excursions. Also, a football match between the teams is often organized (but the results of those matches are not recorded). On the day preceding the departure of the guest team the Competition is concluded with an award ceremony. The delegation of each country at the APMC is headed by two team leaders. The leaders of both delegations constitute the four-person Jury of the Competition. Their task is to select nine problems (from a pool of proposals supplied by both sides), to elaborate the phrasing, translate the problems into the native languages, to read the papers and coordinate the marking, and eventually, to decide on prizes. A prominent mathematician from the host country is invited to chair the final session of the Jury as Honorary Chairman. 1 2 ‘The APMC The scoring is independent in the two contests (team and individual). In the individual contest, the top scorers receive prizes. The number of prize winners varies from year to year, depending on the level and the exact distribution of marks; this number has so far reached its extreme values: five and eight. If two (or more) students score an equal number of points, they occupy the corresponding ranking position ex aequo; so there need not be asingle winner. In the team contest, however, one of the teams has to win. Often it is really difficult to compare their achievements. In such cases it is the role of the Honorary Chairman to make the final decision. Sixteen rounds of the APMC have been “played” already. And since sixteen is a nice round number, I made up my mind to write a compilation of all the problems posed so far. Some of them are very nice and some aré. not; some questions are quite simple, and some are true challenges. I have been proposing problems for olympiads, competitions, contests and problem columns of several journals, for a long time. With each year, Iam more and more convinced about two things. First, it is seldom possible to predict whether a problem will turn out to be easy or difficult to the con- testants. The jumps and meanders of the proposer’s thought process of de- vising a problem lead to a totally wrong image of how difficult the problem is likely to appear to a fresh-minded person who sees it for the first time. Also, the proposer’s “professionalism” can be an additional burden in this respect. The Jury of the APMC has made many serious mistakes in estimating the difficulty of particular problems; the students were the ultimate judges. .. The second point I wish to mention is that it is not at all easy to invent something really new. I have experienced that feeling many times: having composed a problem (which seemed nice and interesting), only to discover, later on, that it had already appeared, many years ago, at some other great competition or in the problem section of some journal. Such was the case with at least one of the APMC problems I am “responsible” for. If the Reader finds more than just one example of that kind in this book, I will not be surprised. Since in the APMC there are only two countries (hence two languages) involved, the Jury attempts to formulate the problems in such a way that the Polish version should be the literal translation of the German version, and vice versa. (A similar endeavour at a competition as large as the IMO would be quite hopeless! Even in the case of these two languages, it often happens that the resulting formulation does not sound very good, either in German or in Polish.) In presenting the APMC problems in this book, I did not feel obliged to produce a third “literal” translation into English; deviations from the original wording do occur without, however, affecting the exact mathematical contents of a problem. ‘The APMC 3 There are given solutions to all the problems. In many cases they are patterned on the proposers’ solution sketches. Many other ones have been inspired by the contestants’ papers; it is not at all rare at olympiads and contests to find that credit for the trickiest and smartest methods has to be given to the students. The presented solution to Problem 10.2 (the true difficulty of which was apparently underestimated by the Jury) is essentially due to Professor A. Schinzel. In several problems, I owe the ideas to people whose names are given in the Acknowledgements. Other solutions are my own contribution. Some solutions are elaborated in full detail; but in most cases routine details are omitted or just briefly outlined. Needless to say, I would never dare claim to have devised the nicest or shortest solutions available. To the contrary, I am quite sure that in many cases you, dear Reader, will find a much more elegant approach. The pleasure and satisfaction will be yours. (The errors you will come across while reading the printed solutions are all mine, of course.) Every (or almost every) problem is given only one solution. Obviously, in most cases there exists a diversity of methods, of which there is no mention here. Try your forces; choose the method of attack you like best. Occasionally a solution is accompanied by a Remark, in which an al- ternative method or a generalization is mentioned. Sometimes the remark is intended as a “box from the tool-chest”: a brief exposition of a solving method that goes slightly beyond the usual school curriculum. ‘The numbering of problems in this book is done according to the follow- ing rule: problem n.k deciphers as the k-th question at the n-th Competi- tion. And thus e.g. problems 13.7, 13.8, 13.9 come from the team contest of APMC-13 (1990, Poland). To end this somewhat lengthy Introduction, I wish all the Readers joy, fun and pleasure in tackling the problems. And, if sometimes you feel angry with either the problem or the presented solution, or your own solution — well, take it easy, that’s mathematics! Marcin BE. Kuczma Institute of Mathematics University of Warsaw Poland PROBLEMS 3 PROBLEMS 1.1. Determine all functions f: Rt —+ R which satisfy f(et+y)=f(e?+y*) forall eye Rt. (R+ denotes the set of all positive real numbers.) 1.2. A parallelogram is inscribed into a regular hexagon so that the centers of symmetry of both figures coincide. Prove that the area of the parallelogram does not exceed 2/3 the area of the hexagon. 1.3. Prove that ‘Vien tan tan < v3 1 < SEE ttn tnt 1.4, Let c # 1 be a positive rational number. Show that it is possible to partition N, the set of positive integers, into two disjoint nonempty subsets A, B so that 2/y # ¢ holds whenever and y lie both in A or both in B. 1.5. There are given 1978 sets, each containing 40 elements. Every two sets have exactly one element in common. Prove that all 1978 sets have a common element. 1.6. We are given a family of discs in the plane, with pairwise disjoint interiors. Each disc is tangent to at least six other discs of the family. Show that the family is infinite. 1.7. Let M be the set of all lattice points in the plane (i.e. points with integer coordinates, in a fixed Cartesian coordinate system). For any point P =(z,y) € M we call the points (x —1,y), (7 +1,y), (2, y—1), (y+) neighbors of P. Let S be a finite subset of M. A one-to-one mapping f of S onto S is called perfect if f(P) is a neighbor of P, for any P € S. Prove that if such a mapping exists, then there exists also a perfect mapping g: S —+ § with the additional property: g(9(P)) = P for P€ S. 1.8. For any positive integer k consider the sequence an = kt Vk Vk (n square-root signs). (a) Show that the sequence converges, for every fixed integer k > 1. (b) Find & such that the limit is an integer. Furthermore, prove that if k is odd then the limit is irrational. 1.9. In a convex polygon P some diagonals have been drawn, without intersections inside P. Show that there exist at least two vertices of P, neither one of them being an endpoint of any one of those diagonals. 7 8 PROBLEMS 2.1 ~ 2.9 2.1. On sides AB and BC of a square ABCD the respective points E and F have been chosen so that BE = BF. Let BN be the altitude in triangle BCB. Prove that DNF is a right angle. 2.2. Determine all polynomials of the form Pp(a) = nla” + dq_ya"-! + -----aye + (-1)"(n+1)n with integer coefficients, having n real roots 71,...,2n which satisfy the inequalities k < ze 3). Each cell can be occupied by one of the two symbols 0,1. The following operation is admissible. Choose any cell C’ occupied by a 1, change it into a 0 and simultaneously change the symbols z,y in the two cells adjacent to C to their complements 1 — 2, 1 —y. In the initial configuration there is a 1 in one cell and Os elsewhere. For what values of n is it possible to obtain Os in all cells, in a finite number of admissible steps? 4.5. Let P(x) = 24+ az + agx? + asx + a4 be a polynomial with rational coefficients. Show that if P(z) has exactly one real root €, then € is a rational number. 4.6. The sequences of real numbers (tn), (Yn), (zn) are given by 1 1 1 Tat =Ynt—, Ynti=%nt+—, 2nd =ln+— for n>0; tn Ln Yn Zo, yo, Zo are given positive numbers. Prove that (zn), (Yn), (zn) are unbounded sequences. 4.7. Let a > 3 be an odd integer. Prove that for every positive integer n the number a” — 1 has at least n +1 distinct prime divisors. PROBLEMS 4.8 - 5.6 11 4.8. The plane has been partitioned into N regions by three bunches of parallel lines. What is the least number of lines needed in order that N > 1981? 4.9. For a function f:[0,1] —+ [0, 1] define the sequence of its iterates by P=fe), PM@)=F(F(@)) for 2€[0,1], n=1,2,3,.... Given that |f"(z) — f*(y)| < |e — yl for a certain n and all z,y € [0,1], x # y, prove that there is a unique zo € [0,1] such that f(xo) = zo. 5.1. Find all pairs of positive integers n,m with ged ((n +1)" —n, (n+ 1)™#3—n) > 1. 5.2. Let C be a circle with center O and radius 1, and let F be a closed convex region inside C. (This means: if P,@ are points of F then the segment PQ is contained in F; all boundary points of F belong to F; all points of F belong to the open disc limited by C.) Furthermore, assume that from each point of C we can draw two rays tangent to F and making an angle of 60°. Prove that F is the disc centered at O, with radius 1/2. 5.3. Let n > 2 be an integer. Prove the equality Ten(5(0+ =5)) . Teo($(1- #4): = 5.4. For a positive integer z let P(z) denote the product of all digits of (in decimal system). Let 2 be any positive integer. Consider the sequence (tq) defined recursively by tn41 = 2n+ P(2q). Prove or disprove that (tq) is necessarily a bounded sequence. 5.5. Suppose that the closed interval [0,1] has been partitioned into two disjoint subsets A, B: AUB=[0,1], ANB=0. Show that there exists no real number a such that B = A +a, where by definition A+a={y: y=2+a, 2€ A}. 5.6. Let a be a given integer. Find all real-valued functions f(z) defined on integers x > a, satisfying the functional equation S(e +) = f(2)f(u) for all z,y such that c>a,y>a,2+y>a. 12 PROBLEMS 5.7 - 6.5 5.7. Find the triple of positive integers (x,y,z), with z least possible, for which there exist positive integers a, b, c, d with the following properties: (i) a aabact ce, g@>a>c; (ti) z=ab=cd; (it) r+ y=atb. 5.8. Let ABCD be the regular tetrahedron with edge length 1 and let P be a point inside ABCD. Show that d(P, AB) +d(P, AC)+4(P, AD)+d(P, BC) +d(P, BD) +d(P,CD) > 3V2, with equality only for P being the centroid of ABCD ; the symbol d(P, AB) denotes the distance from P to line AB. 5.9. Let Sa => a . Find a positive constant C such that = + jae the inequality n 3. Note. The smaller C, the better the solution. 6.1. Nonnegative real numbers a, 6, 2, y satisfy a® +55 <1, 25+y> <1. Show that a223 + b?y3 < 1. 6.2. Find all triples (p,9,n) with p,q prime integers, n > 1 an integer, satisfying P(p +1) +9(9 +1) =n(n+1). 6.3. A bounded planar region of area S is covered by a finite family F of closed discs. Prove that F contains a subfamily consisting of pairwise disjoint discs, of joint area not less than $/9. 6.4. The set N of positive integers has been partitioned into two disjoint sets A, B, their union exhausting all of N. Show that for every n € N there exist distinct integers a,b > n such that the three-element set {a,b,a +b} is contained either in A or in B. 6.5. Let a; < a2 < ag < ay be given positive numbers. Find all real values of parameter c for which the system of equations 2 +29 +2g+24=1 Qy2, + agt +agt3+ayz,=c ajay +032, + a3z5 + a3r4 = c? has a solution (21, 22,73, 24) in nonnegative real numbers. PROBLEMS 6.6 ~ 7.3 13 6.6. There are given six straight lines in the space. In every triple of those lines, two lines are perpendicular. Show that the given lines can be labeled /,...,/s in such a way that 11, lp, ls are pairwise perpendicular, and so are [4, 15,16. 6.7. Let P,, Po, Ps, Ps be four distinct points in the plane. Suppose Ii, Iz, Is, I4, Is, Ig are segments in the same plane, with the following property: every straight line passing through at least one of the points P; meets the union J; UJ2UJ3UJ4U Js UJ in exactly two points (we agree that the endpoints of each J; belong to Ij). Prove or disprove that the segments J necessarily form a hexagon. 6.8. (a) Prove that (2"#! — 1)! is divisible by [[(2"#!~ — 1), for every natural number n. i=0 (b) Define the sequence (¢n) by c1 = 1, cn = ((4n—6)/n) cps for n > 2. Show that each cn is an integer. 6.9. To each side of the regular p-gon of side length 1 there is attached a 1x k rectangle, partitioned into k unit cells; & and p are given positive integers, p and odd prime. Let P be the resulting nonconvex star-like polygonal figure, divided into kp + 1 regions (kp unit cells and the p-gon). Each region has to be colored in one of three colors, adjacent regions having different colors. Furthermore, it is required that the colored figure should not have a symmetry axis. In how many ways can that be done? 7.1, In tetrahedron ABCD, the foot of the altitude dropped from each vertex coincides with the incenter of the opposite face. Show that the tetrahedron is regular. 7.2. Let A be the set of all four-digit natural numbers (in decimal system), written with use of exactly two distinct digits, neither one of them being zero. Interchanging the digits of a number n € A we obtain another number from A, which we denote by f(n). (E.g., f(3111) = 1333.) Determine the number n € A with n > f(n) such that the greatest common divisor of n and f(n) is a maximum. 7.3. Let n > 2 be an integer and let a, 71,...,_ be positive real numbers. Prove that ati-t2—gta—z3 ata— n? ——Ur—t—s—NSOOSsSesSsS and find conditions for equality. 14 PROBLEMS 7.4 ~ 7.9 7.4. AyAzAaAqAsAoAr is the regular heptagon and P is a point of its circumcircle, lying on the minor are A7A;. Show that PA, + PA3 + PAs + PAz = PAz + PAg + PAs. . Let n > 3 be an integer and let ay,...,a, be given nonnegative integers, pairwise distinct. Find all (n +1)-tuples of nonnegative integers (21,...,2n,y), with the greatest common divisor of 21,...,2, equal to 1, such that 2, +42t2+---+ Gnt_ = yt 92, +a3tg+---+ G12, = yr2 ant, + ato + tanith = YEn- 7.6. In a folk dance, the dancers are standing in two rows, n boys facing n girls. Each dancer gives her/his left hand to the person opposite or to her/his left neighbor or to the person opposite the left neighbor. The analogous rule applies to right hands. Nobody gives both hands to the same person. Find the number of possible configurations. 7.7. We are given a rectangular array ay An Gmi ++ Oma of real numbers with |a;;| < 1, such that each column sum ay; + +--+ Gms equals 0. Prove that by rearranging the elements in each column one can obtain an array bu bin bmi v6. Benn such that each row sum satisfies |bi +--+ + bin] < 2 7.8. Consider the functions fole) =22, fi(v) = so for z>l1. Prove that for every real numbers a, b with 1 < a < b there exist an integer k>1 and a k-tuple of indices i1,i2,...,i% € {0,1} such that a< fi, (Fy_y(--- (fn ())--)) 8 for which this is possible. (It is understood that “to know” is a symmetric nonreflexive relation: if P; knows P; then P; knows P;; to say that P; knows p persons means: knows p persons other than himself/herself.) b 8.3. In a convex quadrilateral of area_1, the sum of the lengths of all sides and diagonals is not less than 4 + V8. Prove this. 8.4. Find all pairs of real numbers 2, y satisfying the system of equa- tions ri+y?- ay - 32=0, yf +2? — oy — gy =0. 8.5. We are given a certain number of identical sets of weights; each set consists of four different weights expressed by natural numbers (of weight units). Using these weights we are able to weigh out every integer mass up to 1985 (inclusive). How many ways are there to compose such a set of weight sets given that the joint mass of all weights is the least possible? 8.6. Let P be a point inside a tetrahedron ABCD and let Sa, Sp, Sc, Sp be the centroids (i.e. centers of gravity) of the tetrahedra PBCD, PCDA, PDAB, PABC. Show that the volume of tetrahedron S4SpScSp equals 1/64 the volume of ABCD. 8.7. Find an upper bound for the ratio @122 + Weg + gr4 ap+2}+23+23 over all quadruples of real numbers (21,22, 23,4) # (0; 0, 0,0). Note. The smaller bound, the better the solution. 8.8. A convex n-gon ApAi...An-1 has been partitioned into n — 2 triangles by certain diagonals not intersecting inside the n-gon. Prove that these triangles can be labeled Aj, A2,...,An—2 in such a way that Aj is a vertex of Aj, for i= 1,...,n —2. Find the number of all such labelings. 16 PROBLEMS 8.9 - 9.9 8.9. We are given a convex polygon. Show that one can find a point Q inside the polygon and three vertices Ai, Az, Ag (not necessarily consecu- tive) such that each ray A;Q (i = 1,2,3) makes acute angles with the two sides emanating from Aj. 9.1. We are given a non-right-angled triangle AyAzAs. Points Oi, Oo, Og are the centers of circles I, I, Is, pairwise tangent (internally or externally), circle [, passing through Az and Ag, circle I passing through Ag, A1, circle I's passing through Ay, Az. Given that the triangles A, A243 and 010203 are similar, determine their angles. 9.2. The monic polynomial P(z) = 2" + a,—12"-! + +--+ aye + a9 of degree n > 1 has n distinct real negative roots. Prove that a, P(1) > 2n?ap. 9.3. Each point in space is colored either blue or red. Show that there exists a square with edge length 1, having exactly 0, 1 or 4 blue vertices. 9.4. Find all triples of positive integers (z, y, 2) such that gtth = ytHt = 9100, 9.5. Find all quadruples of real numbers (2, y, u, v) satisfying the system of equations e+pPtu¢vr=4 cutyo+evut+yu=0 cyutyuv+uve+ury = —-2 zyuv =—1. 9.6. Let M be the set of all tetrahedra whose inscribed sphere and circumscribed sphere are concentric. Denoting the radii of these spheres by r and R respectively, determine the range of possible values of the ratio R/r, over all tetrahedra of class M. 9.7. Let k and n be integers with 0 < k < n?/4. Assume that k has no prime divisor greater than n. Prove that n! is divisible by k. 9.8. Pairwise distinct real numbers are arranged into a rectangular array with m rows and n columns. In each row the entries are arranged increasingly (from left to right). Each column is then rearranged in de- creasing order (from top to bottom). Prove that in the reorganized array, all rows are again arranged increasingly. 9.9. Determine all continuous monotonic functions f: R —+ R which satisfy f(1)=1and f(f(z)) = (f(z)? forall ze R. PROBLEMS 10.1 - 10.7 17 10.1. Three pairwise perpendicular chords of a sphere S are drawn through a given point P inside S. Show that the sum of the squares of their lengths does not depend on their directions. 10.2. Let n be the square of an integer, whose every prime divisor has an even number of digits in its decimal representation. Let P(z) = 2” —1987z. Prove that, for rational numbers z,y, the equality P(z) = P(y) implies gay. 10.3. Let f: R —+ R be a function satisfying f(r +1) = f(z) +1 for z € R and let a € R be a fixed number. Define the sequence (zn) recursively by to = a, Zny1 = f(2n) for n > 0. Suppose there exists a positive integer m such that the difference zm — zo equals k, also an integer. Show that the limit lim(z,,/n) exists; express its value in terms of the given data. 10.4, Does the set {1,2,...,3000} contain a subset A consisting of 2000 numbers such that z € A implies 22 ¢ A? 10.5. The Euclidean three-dimensional space has been partitioned into three nonempty sets Ai, 42, Aa. Show that one of these sets contains, for every positive number d, a pair of points at mutual distance d. 10.6. Let C be the circle with radius 1 and let n > 1 be a fixed integer. For any set A of n points P1,..., Pn on C define D(A) = max(mind(P:,d)) where 6(P, 1) denotes the distance from point P to line | and the maxi- mum is taken over all diameters d of circle C. Let F,, be the family of all n-element subsets A C C and let Dn = min D(A). AEF y Calculate D, and describe all sets A € F, with D(A) = Dn. £ 10.7. For any natural number n= }_a;10' (where a; € {0,1,...,9}, a, # 0) write i=0 . . POM gr) ee Bier ee ac) I = #50 Consider the set P={n: n=n", p(n)/3= s(n) —1} and let Q be the set of all numbers in P with all digits greater than 1. (a) Show that P is infinite. (b) Show that Q is finite. (c) Write down all the elements of Q. 18 PROBLEMS 10.8 ~ 11.5 10.8. The circle of length 1 has been dissected into four arcs By, Ba, Bs, Bg, each of length 1/4. A closed non-selfintersecting contour C’ has been composed of (translates of) these arcs (each B; possibly occurring several times). The curve C is smooth; this means that every two adjacent arcs are joined in one of two ways, either to form a semicircle or a wave-like line (a tilde); the arcs have common tangents at junctions and make no cusps. Prove that the length of C is an integer. 10.9. Let M be the set of all points (x,y) in the plane, with integer coordinates satisfying 1 < 2 < 12, 1 < y < 13 (in a fixed orthogonal coordinate system). (a) Prove that every 49-element subset of M contains four vertices of a rectangle with sides parallel to the coordinate axes. (b) Give an example of a 48-clement subset of M without this property. 11.1. Let P(z) be a polynomial with integer coefficients. Show that if Q(z) = P(z) +12 has at least six distinct integer roots, then P(x) has no integer roots. 11.2. Let a1, that the inequality yay, (n > 2) be integers with 1 0 ra = holds for every n-tuple of real numbers (21,...,2n) # (0,...,0) if and only if az > 2. 11.3. Let ABCD be a convex quadrilateral (with all interior angles less that 180°), having no pair of sides parallel. Consider the two angles formed by two pairs of opposite sides, Their (internal) angle bisectors intersect, the four sides of ABCD in points P, Q, R, S, so that PQRS is a convex quadrilateral. Show that ABCD has a circumcircle if and only if PQRS is a rhombus. 11.4. Determine all strictly increasing functions f: R — R satisfying the functional equation f(F(z) +) =F(e+y)+F(0) forall 2,yeR. 11.5. Two infinite sequences of integers, (az) and (bg), are related by be=leag+9, pyr =8-be+8 for k>D0. Suppose that the number 1988 appears either in the sequence (az) or in (bx). Show that the sequence (ay) does not contain a perfect square. PROBLEMS 11.6 ~ 12.4 19 11.6. Given are a point O and three non-coplanar rays hi, ho, hs emanating from O, such that for every triple of points A, € hi, Az € ha, ‘As € ha (A: # O, # = 1,2,8) the triangle A, A2As is acute-angled. Show that hi, ho, hg are pairwise perpendicular 11.7. In a regular octagon each side is colored blue or yellow. From such a coloring a new coloring is obtained in one step (“move”) as follows: if the two neighbors of a side have different colors, the new color of that side will be blue, otherwise it will be yellow (the colors are modified simultaneously). Show that after finitely many moves all sides will be colored yellow. What is the maximum number of moves that may be needed to achieve this state? 11.8. We are given 1988 congruent cubes (of edge length 1). Using some or all of these cubes we form (“glue”) three quadratic boards A, B, C of side lengths a, 6, c with a 1) of squares of positive integers such that for each n the difference a,41— dn is a prime or the square of a prime. Show that all such sequences are finite and determine the longest, one. 12.7. Functions fo, fi, fo... are defined recursively by folz)= 2 for ER, forss(z)=37*™) for ER, k=0,1,2,..., fox(z)= V2 for 2ER, k=1,2,3,.... Determine, with proof, which one of the two numbers fio(1) and fo(2) is greater. 12.8. We are given an acute triangle ABC. For each point P of the interior or boundary of ABC let Pa, Ps, Pz be the feet of the perpendiculars dropped from P to sides BC, CA, AB, respectively. Define _ AP. + BPa+CP, MP) = BPt PP + PP Prove that f(P) is constant as P varies if and only if the triangle ABC is equilateral. 12.9. Determine the smallest odd N such that N? is the sum of an odd number (greater than 1) of squares of successive positive integers. 13.1. Let A, B, Pi, Po, ..., Ps be eight distinct points in the plane, the Pjs all lying on the same side of line AB. Suppose the six triangles ABP, (1 1 consider the system of equations af +422 +1= yf 4+ l42.23+1= yf ait Monti +1= yf. Find all solutions (21,...,2n, Y1s---,Ya) with 2; and y; (for 1 < i < n) being positive integers. 13.5. Let n > 1 be an integer and let 5, be the set of all permutations (one-to-one maps) 7:{1,...,n} —> {1,...,n}. For every permutation 7 € Spy define F(n)= ye nk) and My = a yo Fr) 7ESn (summation ranging over all permutations € Sy). Compute Mn. 18.6. Let P(z) be a polynomial with integer coefficients. Suppose that the integers 21, 22, ..., Zn (n > 3) satisfy the conditions P(z;) = 2:41 for nee ey Show that 21 = 23. 13.7. Given is a set S, of domino pieces {Ojo}, [012], ---, [OlnJ, (22), [212], --. 5 (for each pair a,b with 0 < a < b < n there is exactly one piece [ald], denoted also by the symbol [b|a]). By a chain we mean any sequence of pieces in succession (a1 |a2][a2|a3} .. .[ax—2lax—1][axe-s|ax] (e.g. {015)(5|5]{5|2]{1|2] ). The chain is closed when ay = ay. (a) If n is even, prove that there exists a closed chain composed of all pieces. (b) If n is odd, prove that every closed chain leaves at least (n + 1)/2 pieces unused. (c) Assume n is odd. How many sets A are there, each consisting of exactly (n + 1)/2 pieces, such that all pieces that do not belong to A can be arranged into a closed chain? 22 PROBLEMS 13.8 - 14.4 13.8. Let R be a 28 x 48 rectangle. We are concerned with dissections of R into congruent a x b rectangles (a # 6, a,b € N) with sides parallel to the sides of R. For some pairs (a,b) there exist several such dissections, for some (a, 6) just one. (a) Determine the sides a, b of the rectangle with the smallest area for which the dissection of R into a x 6 rectangles is unique. (b) Determine the sides a, 6 of the rectangle with the largest area for which the dissection of R into a x 6 rectangles is not unique. (Dissections which are not identical are considered to be different, also in the case when one of them results from the other one by rotation or symmetry.) 13.9. Let ai,a2,...,a, be integers such that every “partial” sum aj, + a;, +-+--+ aj, is different from zero (1 < i: < ip < +++ < ie 2 such that the equality m n (2) =*(@) holds for some integer n = n(m) > 4. Give the general form of all such m. 14.2. Find all triples of real numbers (2, y, z) satisfying the system of equations (y? — 6y +:13)z (x? — 6x + 13)y= 20 (2? 62+ 13)2 14.3. Given are two distinct points Ay, Az in the plane. Determine all possible positions of a point As with the following property: There exist an integer n > 3.and n points P,, P2, ..., Pa (not necessarily distinct) such that the segments P;P2, P:P3,..., Pn—1Pn, PnP; have equal lengths and their midpoints are A1, Az, As, A1, A2, As, -.., in this order. 14.4. Let P(2) be a real polynomial with P(x) > 0 for 2 € [0,1]. Prove that there exist polynomials P;(z) (i = 0,1,2) with P;(2) > 0 for all real and such that P(x) = Po(x) + eP,(x) + (1 — 2) P2(z). PROBLEMS 14.5 ~ 15.3 23 14.5. Prove the inequality ayer taytys te > 2UJSet Vyt Vz) for every positive numbers z, y, z with eyz = 1. 14.6. Inside a convex quadrilateral ABCD (with all internal angles less than 180°) there is a point P such that the triangles PAB, PBC, PCD, PDA have equal areas. Prove that the area of ABCD is bisected by one of the diagonals. 14.7. For a given integer n > 1 determine the maximum value of the function cterge pamet f(@) = ——Gyaay over z € (0,00) and find all 2 > 0 for which the maximum is attained. 14.8. Consider the system of simultaneous congruences zy=-1 (mod z) 1 (mod z) z= 1 (mody). Find the number of triples (2, y, z) of distinct positive integers satisfying this system and such that one of the numbers z, y, z equals 19 14.9. Let A = {1,2,...,n} with n a positive even integer. Suppose that g: A—> A is a function with g(k)#k and —g(g(k)) =k for KEA. How many functions f: A —> A are there such that S(k) #a(k) and —f(F(F())) = 9k) for KE A? 15.1. Let o(n) denote the sum of all positive divisors of a natural number n > 1. Prove that for every natural number n > 1 the product a(n — 1)o(n)o(n + 1) is even. 15.2. Each point on the boundary of a square has to be colored in one color. Consider all right-angled triangles with three vertices on the boundary of the square. Determine the least number of colors for which there exists a coloring such that no triangle (considered) has all its vertices of equal color. 15.3, Prove the inequality 2Vocteatab< V3 Y(b+ej(et aj(ato) for positive real numbers a, b, ¢. 24 PROBLEMS 15.4 ~ 18.8 15.4. Let k be a natural number and u, v be real numbers. Set P(z) = (2 -u*)(x — uv)(z -— v*) = 2 + an? +b2te. (a) Let k = 2. Prove: if a, 8, ¢ are rational, then the product uv is rational. (b) Is that true also for k = 3? 15.5. Given is a circle I’ with center O and radius r. Let AB be a fixed diameter of I” and let K be a fixed point of segment AO. Denote by t the line tangent to [' at A. For any chord CD (other than AB) passing through K let P and @ be the points of intersection of lines BC and BD with t. Prove that the product AP- AQ remains constant as the chord CD varies. 15.6. Let Z be the set of all integers. Consider a function f: Z —> Z with the properties: (92 + 2) = (92-2) f(19-92+2) = f(19-92—z2) (19-92 = 1748) f(1992 + 2) = (1992 - 2) for all z € Z. Is it possible that all positive divisors of 92 occur as values of f? 15.7. We are considering triangles ABC in space. (a) What conditions must be fulfilled by the angles a, 8, 7 of triangle ABC in order that there exists a point P in space such that ZAPB, BPC, LCPA are right angles? (b) Suppose there exists a point P with the property stated in (a); let d be the maximum distance among PA, PB, PC and let h be the longest altitude of triangle ABC . Show that (V6/3)h 3 be a fixed integer. Assume that the real numbers @),...,n (different from zero) satisfy the equations Tai = ag + ages +Gn _ Gy ~~ agtagte tan _ a _ a 2 ate + On-2 = nw — On . Qn-1 = Data tet an-1 — an Gn What values can be taken (for the given n) by the product, +an PROBLEMS 15.9 ~ 16.6 25 15.9. Let n > 1 be an integer. We are considering words composed of n characters A and n characters B. ‘A word X;...Xaq is said to belong to class R(n) if no initial segment Xz (1 < k < 2n) consists of equally many As and Bs. A word ...Xaq is aid to belong to class S(n) if exactly one initial segment X1...Xp (1< k < 2n) consists of equally many As and Bs. Suppose there are r(n) words in class R(n) and s(n) words in class S(n). Compute the ratio (n)/r(n). ce, 16.1. Solve the equation 2° — 3” =7 in natural numbers z,y > 1. 16.2. We are considering all tetrahedra ABCD such that the sum of areas of faces ABD, ACD, BCD does not exceed 1. Find all those tetra- hedra, in the class under consideration, that have the maximum volume. 16.3. For a natural number n = p* > 1 being a power of a prime, set f(n) =n41. For n = pi!---prt (r > 1) being a product of powers of several distinct primes, set f(n) = pi! +-+--+ps". For each m > 1 consider the sequence ag=m, ays = f(aj) for 720 and denote by g(m) the least term of this sequence. Find the value of g(m), for every integer m > 1. 16.4. The Fibonacci numbers are defined recursively by ge eee Fide Ones ea a hee for. a> i0 Let A and B be positive integers such that A!® divides B®, and B® divides A®?. Show that, for every natural number n > 1, the number (A4 + B®) "+1 is divisible by (AB). 16.5. Solve the system of equations { B+ya 3044 in real numbers 2, y, z. 16.6. Prove the following three inequalities for real numbers a,b > 0: (4ety < at Vat + Yat? +b 2 i 4 at vab+b - ( ey ~ 3 ~ 2 For each one of these three inequalities, find out when equality holds. 26 PROBLEMS 16.7 ~ 16.9 16.7. The sequence ao, a1, az, ... is defined by ao=0, angi =[Yantn]® for n>0 ( [z] denotes the greatest integer not exceeding z). (a) Give an explicit formula for an, in terms of n alone. (b) Find all n such that a, =n. 16.8. Find all real polynomials P(z) for which there exists a unique real polynomial Q(z) with Q(0) = 0, satisfying the equation 2+Q(y+P(2))=y+Q(z+P(y)) for tye R. (To distinct polynomials P(z) there can correspond distinct polynomials Q(z)-) 16.9. Let ABC be an equilateral triangle with side length a and let P be a point on line BA produced beyond A. Let r; and ry be the radii of the incircle of triangle PAC and the excircle of triangle PBC, touching side BC and lines PB, PC produced. Calculate the sum r; + r2 27 SOLUTIONS => 28 SOLUTIONS 1.1 Determine all functions f: R+ —+ R. which satisfy f(e+y)=f(2? +y") forall z,yeR*. (R* denotes the set of all positive real numbers.) Solution Suppose f satisfies the condition of the problem. Choose positive num- bers a, b with a < b < 2a and set 2=(Va-Ve—a)/V2, y= (Va+Vb—a)/V2. Evidently, z,y > 0, 2 +y = V2a, 2? +y? = b, and hence f(a) = f(b). Keeping a fixed and letting b vary over [a, 2a), we see that f is constant in this interval. This is valid for any choice of a, so f must be constant in every interval of the form [a,2a). Now, if u,v are positive numbers, u < v, then we can find finitely many successively overlapping intervals of the type [a, 2a) so that u belongs to the first and v belongs to the last of them. Consequently, f(u) = f(v), and hence f is constant on Rt. ‘And conversely, it is plain that every constant function satisfies the given equation. 1.2 A parallelogram is inscribed into a regular hexagon so that the centers of symmetry of both figures coincide. Prove that the area of the parallelogram does not exceed 2/3 the area of the hexagon. Solution G F E H Let ABCD be the parallelogram and let E be a vertex of the hexagon whose distance from line AC is a maximum; clearly, the vertex opposite to E, denote it by G, is equally distant from that line. The atea of parallelo- gram AEGG is not less than that of ABCD. Repeating the argument with respect to line EG, we find another pair of opposite vertices of the hexagon, 29 30 SOLUTIONS 1.2 - 1.3 F and H, so that the area of EFGH is not less than that of AECG (in fact, these areas are equal). Finally, note that BFGH is a rectangle of area exactly 2/3 the area of the hexagon. Hence the claim. 1.3. Prove that ‘Vian 1° -tan2°---tan44° < V2-1 tan 1° + tan 2° +--+ tan 44° < 44 Solution Both parts follow easily from the Jensen inequality; see the Remark (below). The function f(z) = tanz is strictly convex and g(z) = Intana is strictly concave in the interval (0, 7/4); to see this, examine the derivatives: f'(2) = (cos z)-? is increasing and g'(z) = 2(sin2z)~! is decreasing there. Writing 2, = kn/180 (the radian measure of the angle of k degrees) we thus get Eee CT) > (24 ti) =5(2), ay se) +t alee) < (24+ 24) = (3). (2) Now, z = c08(n/4 Be wi He #(§) = tan "= an(r/ay 2-4 so (1) is exactly one of the claimed inequalities. The other one follows from (2) by observing that gen) +--+ 988) _ iy sy tana; Tana 44 and z 9(5) =In(v2-1). Remark A real-valued function, defined in some real-line interval, is conver if there is no point C on its graph lying above the chord joining any two other points A and B of that graph such that the abscissa of C is comprised between the abscissae of A and B. A convex function is said to be strictly convez if its graph does not contain any straight line segment of positive length. A function f is (strictly) concave if —f is (strictly) convex. For instance, the functions z?, e” are strictly convex, while \/z, Inz are strictly concave (in their usual domains). SOLUTION 1.3 31 A sufficient condition for convexity or concavity is the existence and monotonicity of the derivative f’: if f” is (strictly) increasing, then f is (strictly) convex; if f’ is (strictly) decreasing, then f is (strictly) concave. A function f which is convex in some interval I, satisfies the Jensen inequality F(qiti + +++ + dn®n) Sf (21) + +++ Inf(tn); (3) for every choice of numbers z;,...,Z, € I and every nonnegative numbers (“weights”) 1,...,9n with gq: +-+-+ Qn = 1. When all weights are equal to 1/n, (3) takes the form (24 +#) < fle+ n If f is concave, the inequalities in (3) and (4) have to be reversed. For n = 2 the inequality (3) becomes: F(qu+(1~4)v) < af(u) + (1 - a) f(v) (5) for any u,v € I and any q € [0,1]. This is just an analytic restatement of the geometric description of convexity we have begun with. For a general n > 2, (3) can be proved by induction. Assume (3) holds with n — 1 in place of n and suppose we are given n numbers 21,...,2, € J and n weights q1,...,gn > 0 (summing to 1). Without loss of generality, assume qi > 0. Define g=q1+-+:+4n-1>0, pi = 9:/q fori =1,...,.n—1, 80 that p) +--+ pn-1= 1. Applying, first, the inequality (5) (which defines convexity), and then the inductive assumption, we obtain F(qiz1 +--+ + Qntn) = f(g(piti + +--+ Pn-1tn-1) + (1 — 9)2n) Saf (piti + +++ + Pn-1tn-1) + (1-9) Fn) << a[pi f(e1)+- + +Pn-1F(tn=1)] +9nF(@n) = af (21) +--+ 9nf (En); completing induction. If f is strictly convex (or strictly concave), the inequality in (4) (or the reversed inequality) is strict unless all the numbers z; are equal. It is worth noticing that the familiar inequalities + f(tn) | (4) n tn +n) for @1,...,4n >0 (6) (the arithmetic mean-geometric mean inequality) and (a1by +-+-+ nbn)” < (a3 +--- +2) (O23 +----402) for a:b; ER (7) (the Cauchy-Schwarz inequality) are simple instances of (3) (or (4)). Set- ting in (4) 2; = Ina;, f(x) = e*, we obtain (6); to deduce (7), use (3) with F(x) = 27, qi = 07/(02 +--- +2), 2; = a;/b; (if some bjs are zero, just ignore them, together with the corresponding ajs). 32 SOLUTIONS 1.4 ~ 1.5 1.4 Let c #1 be a positive rational number. Show that it is possible to partition N, the set of positive integers, into two disjoint nonempty subsets A, B so that 2/y # c holds whenever « and y lie both in A or both in B. Solution This can be done in many ways. Here is a simple construction: Assume without loss of generality c > 1 (if ¢ < 1, replace c by 1/c; this does not affect the conditions of the problem). For every integer k, positive or negative (or zero), denote by J, the interval [c#,c*+1). The union of these intervals is R+ = (0,00). Note that the ratio of two positive real numbers can be equal to c only if they belong to two adjacent [,s. Hence if we define A (respectively, B) to consist of those natural numbers that belong to the union of Izs with even (respectively, odd) indices k, the resulting partition N = AU B will meet the demands. 1.5 There are given 1978 sets, each containing 40 elements. Every two sets have exactly one element in common. Prove that all 1978 sets have a common element. Solution Denote the given sets by Ao, A1, ---; Anj in the case under discussion, n = 1977; we can, however, consider the case of general n, assuming that each A; is a k-element set (k = 40 in our case). To each element 2 € Ao we assign the set of those indices j € {1,...,n} for which x € Aj. Since Ao has k elements and each two of the Ajs have exactly one common element, this defines a partition of {1,2,...,n} into k disjoint classes (some of them may be empty). By the pigeonhole principle, at least one of these classes contains at least m indices where m is the smallest integer > n/k (often denoted m = {n/k]). Relabeling if necessary, we assume that I,...,m belong to a common class; that is, there exists an element ro € Ap Ai... Am- ‘We will find conditions on n and k which ensure that then zp must be a common element of all the given sets. Suppose this is not the case; assume e.g. 20 ¢ An (again relabeling if necessary). Each A;, 0 k? — k and let Ao,...,An be k-element sets, every two having exactly one common element; then there exists an element common to alln +1 sets. In particular, this is the case for n = 1977, k = 40. SOLUTIONS 1.6 ~ 1.7 33 1.6 We are given a family of discs in the plane, with pairwise disjoint interiors. Each disc is tangent to at least six other discs of the family. Show that the family is infinite. Solution Assume that the family is finite and let Do be the disc with smallest radius. Do touches at least six discs Dy,...,Ds. Let O; be the center of D;, i=0,1,...,6. We may assume that D,..., Dg are arranged cyclically around Dp, in the sense that the wedges 01002, O2000a, ..., Os0001, 01000> are successively adjacent. Segment 010: is the longest side of triangle Oo0;02; therefore Z0,0o02 > 60°. Similarly 40;0o0;41 > 60° for i = 1,...,6 (where Or = O;). The sum of sizes of these six angles is equal to 360°. Consequently 40;0oO;41 = 60° (i = 1,...,6) and the triangles 0;000i41 are equilateral. Say So Dy is another disc of minimal radius. The same reasoning shows that it is surrounded by six other discs (of the family considered), with the same radius. The center of one of them is the symmetric image of Oo with respect to Oy. Continuing, we find a sequence of points on line Qo01, at equal distances, each of them being the center of some disc of the family. ‘The procedure never terminates. The family cannot be finite. 1.7 Let M be the set of all lattice points in the plane (i.e., points with integer coordinates, in a fixed Cartesian coordinate system). For any point P = (z,y) € M we call the points (z —1,y), (r+ 1,y), (z,y — 1), (e,y +1) neighbors of P. Let S be a finite subset of M. A one-to-one mapping f of S onto S is called perfect if f(P) is a neighbor of P, for any P € S. Prove that if such a mapping exists, then there exists also a perfect mapping g: S —+ S with the additional property: g(g(P)) = P for PES. Solution Let So, respectively 51, be the set of all points (zy) in S with 2+y even, respectively odd. Note that every two neighboring points in $ belong one to So and the other to S, (visualize M as an infinite chessboard, with 34 SOLUTIONS 1.7 ~ 1.8 white and black cells). Thus a perfect mapping f: S —+ S maps So onto S; and vice versa. Let f-! be the inverse mapping of f. We can define g as follows: f(P) if PE Se, OS {feo ifP eS, Obviously, g is a one-to-one mapping, sending each point of $' to one of its neighboring positions in S. Therefore g is perfect. It is also evident from the definition of g that g(g(P)) = P, as required (a mapping with this last property is said to be involutive). Note that the finiteness of S does not play any role. 1.8 For any positive integer k consider the sequence an kt+Vk Vk (m square-root signs). (a) Show that the sequence converges, for every fixed integer k > 1. (b) Find & such that the limit is an integer. Furthermore, prove that if k is odd then the limit is irrational. Solution (a) The sequence is defined by the recursion — iéé=eiOEF§SFf We show by induction that an On: It is obvious for n = 1. Assume the inequality holds for some n. Then ang2 = Vk + angi > Vk + an = Oni, completing the inductive step. So (an) is an increasing sequence, bounded above, hence convergent. (b) Denote the limit by 1. The recursion formula a2, = k + an yields, in limit, 1? = k +1. Since all as are nonnegative, so is 1. Thus / is the nonnegative root of the equation z?—- z—k = 0; i.e., t= 4(14+ V4k+1). SOLUTIONS 1.8 ~ 1.9 35, Taking for & a number of the form m? +m, where m is a nonnegative integer, we obtain | = m +1, an integer also. If k is odd then 4k-+1 is 5 (mod 8) and cannot be a perfect square (every odd perfect square equals 1 modulo 8). Thus V4k +1 is not an integer. The square root of a natural number is either an integer or an irrational number. Consequently / is irrational. 1.9 Ina convex polygon P some diagonals have been drawn, without intersections inside P. Show that there exist at least two vertices of P, neither one of them being an endpoint of any one of those diagonals. Solution The distinguished diagonals dissect P into a-finite number of polygonal regions. Each side of any of these regions is either a side of P or one of the given diagonals. Let Ro,..., Rn be a chain of distinct, successively adjacent regions, of maximal length; that is to say, Rj, and R; (i = 1,...,n) are assumed to have a common side (which is necessarily one of the diagonals in question), and there exists no chain of more regions with this property. We claim that only one of those diagonals makes a part of the boundary of Rn. For suppose there are more. Then R, would have to be adjacent to some region other than R,-1. Note that R, cannot be adjacent to any Ry with 0 < i n!. The resulting inequality n +1 > n!(n— 1)! is satisfied only when n = 1 or n=2 Forn=1, P,(x) = x— 2; the (unique) root x; = 2 belongs to [1,2], as required. For n = 2, Po(x) = 227+ a12+6. The roots 11,22 have to satisfy legm<2 6] a: fal it tad isi holds for every n-tuple of positive numbers (a1,...,an)- Solution Suppose that n has the demanded property. Setting, first, a; = 1 for i=1,...,n, and then a; =n for i=1,...,n, we obtain the following two inequalities: n?—n>6 and n—1>6n"4 The first of them fails to hold for n < 2, the second fails for n > 4. Hence n=3. When n = 8, the obvious inequality (a2 — a3)? + a2(a3 ~ a1)? + a3(a1 — a2)? > 0 after slight recasting produces exactly the inequality in question. So n = 3 is the answer to the problem. 2.4 Determine all real-valued functions f, defined on nonnegative in- tegers, satisfying fe +) + fey) = f@z). Solution Setting for (x,y) the pairs (t,0), (4,1) and (2¢,4), with an arbitrary ¢, we obtain respectively 2F() = F(3t), F(t) + FO) = F(3t), F(34) + F) = F(t). With 1 = 0, the first of these equations yields f(0) = 0; so the first two equations jointly result in f(2t) = 2/(t). Hence, the third equation implies F(3t) + f(t) = 2f(3t). Thus f(3t) = f(t). Revisiting the first of the three equations above we now get f(t) = 2f(t), for any t, and therefore f must be the constant null function. 2.5 The centers of the circumscribed sphere and the inscribed sphere of a given tetrahedron coincide. Prove that the faces are congruent triangles. 38. SOLUTION 2.5 Solution Let S be the sphere circumscribed about the given tetrahedron ABCD; denote its center by O. Since O is also the center of the inscribed sphere, plane BCO bisects the dihedral angle formed by faces BCA and BCD. The intersection of plane BCA with sphere $ is a circle, the circumcircle of triangle BCA; the intersection of plane BCD with S is the circumcircle of triangle BCD. These two circles are symmetric with respect to plane BCO. Since BC is a common chord of two congruent circles, the angles BAC and BDC (subtended by congruent arcs BC) are equal. Denote their size by a. Quite analogously we match other pairs of equal angles and denote them as follows: LBAC = LBDC =a, LABC =LADC =8, LACB= ADB =7, 4ABD=LACD=9, LBAD=LBCD=4, LCAD=/CBD =u. In every triangle, the angles sum up to 180°. Considering the four faces of ABCD we thus have: in ADCB: = a+P+w = 180°; in ACDA: = y+f+w = 180°; in ABAD: p+ +7= 180°; in AABC a+ B+ y= 180°. Subtracting the first two equations, a— 6 = p—w. Subtracting the last two equations, a+ 8= y+. Hence p=a, y= 6 and w =. Consequently, all the four faces are similar triangles with angles a, f, 7. Considering for instance triangles ABC and DCB, the interior angles ZA, 2B, ZC of the first one are matched respectively with angles 2D, SOLUTIONS 2.5 - 2.6 39 3. Consider the first three equations of the system = 0; hence the pair Byte +2n =a, (a) aite+-+22 =a, (2) apts +28 = a8. (3) If a = 0 then by (2) 2; =--- = 2, = 0. Ifa ¢ 0, equation (2) yields 2? 2 and m > 2, so that the “criti- cal” square Q, is adjacent to two other squares. The situation is slightly different when n = 1 or m= 1, and the hexagon reduces to a rectangle (of size 1x m or nx 1); the number of admissible paths equals 2"+"~! in that case. 2.8 — Given are four points A, B, C, D in space. If M and N are the midpoints of AC and BD respectively, show that AB? + BC? + CD? + DA? = AC? + BD? +4-MN?. Solution Choose any point O and consider the vectors u=04, v=0B, w=0C, 2=OD. Then For any vector x = XY, XY? = (length of x)’ =x+x= where the dot (and squaring) denotes inner product (scalar product). Thus the left side of the claimed equality is AB? + BC? + CD? + DA? = (vu)? + (w—v)? + (2—w)? + (uz)? = 2(u? + v? + w? +27) — -Aurvtvew+wes+zeu), while the right side equals AC? + BD? +4 MN? = (w—u)? + (2—v)? + ((v+2) -(u+w))? = (u? +.v? + w? +2? — Quew — Qve2) + +(v +2)? +(u+w)? —2(v +2) -(u+w) = (u? +-v? +.w? +2? — Quew —2Qvez) + + (u? 4 -v? 4 w? +2? 4 2ven+2u-w) — -—Qurv+vew+wz+zeu) = 2(u? + v? +.w? +27) - —Aurv+vew+weztzeu). ‘The two expressions are equal and the claim is proved. 42 SOLUTIONS 2.9 ~ 3.1 2.9 For a nonnegative integer n let an = [(3+VII)”"**] be the greatest integer not exceeding (3+ VI1)""*7. Find the greatest power of 2 that divides an. Solution If the number (3 + V11)7"** is represented in the form un + taVI1, where un, Un are integers, then (3— VI1)**** equals un — taVI1; this follows immediately from the binomial formula. Thus (3+-Vi1)""** + (3 - VIT)"** = un is an integer; and since the second summand on the left side of this equality is a number between —1 and 0, this expression is exactly the integer part of (34+ Vil)": Qn = Qn. From the equalities ung1 £ UngiV1l = (3+ Vi1)""*? = (3 VIT)? (ua + unV11) = 2un + (6un + 2000p) VII + 660q we obtain the recurrence formulas Unt1 = 20tn + 66un , Ungt = bun + 20%n And since uo = 3, vo = 1, obvious induction shows that n is the highest exponent such that 2” divides both un and tq. Consequently, 2"+? is the highest power of 2 dividing an. 3.1 Three infinite arithmetic progressions are given whose terms are positive integers. Assuming that each of the numbers 1, 2, 3, 4, 5, 6, 7, 8 occurs in at least one of these progressions, show that 1980 necessarily occurs in at least one of them. Solution According to assumption, 6 is a term of a progression (an). If the step d= Gn41 — ay of the progression (an) equals 1, 2 or 3, then 1980 = am for some m (because 1980 is divisible by 6). Thus assume d > 4. Then the numbers 3, 4, 5, 7, 8 must enter the two remaining progressions (bn) and (cn). The case where either (bn) or (cn) has step 1 is trivial. Assume this is not the case. Then 3 and 4 enter distinct progressions, and so do 4 and 5. Hence 3 and 5 are in the same progression, say, (bn), which, therefore, misses all even numbers. So 4 and 8 are in (cn), and consequently 1980 is there, too. SOLUTIONS 3.2 - 3.3 43 3.2 Let 21, 22,a3,... be a sequence of integers such that Lsa, 360°. @ Changing the roles of vertices suitably we have two analogous inequalities (APB + LCPB) + (LAPD + LCPD) > 360°, (2) (LAPB + LDPB) +(LAPC + LDPC) > 360°. (3) Each one of the six angles mentioned in the statement of the problem appears twice in the left-side expressions of (1), (2), (3). Hence LAPB+ZLAPC+ZAPD+ZBPC+4BPD+2CPD> $(3-360°) = 540°. 3.4 Prove that yy =n, iin summation spreading over all nonempty subsets {i1, i2,..., iz} of the set {1,2,...,n}. Solution Consider the product 1 1 1 1 P= (16 DD (4) 049) Multiplying out, we obtain the sum of 2 terms, one of which equals 1 and the others which constitute exactly the sum we wish to evaluate. So it remains to show that P = n+1. But this is obvious: 234 n+1 Dp pe antl 3.5 Let Aj A2Az3 be a triangle and let B,, Bz, Bs be points on sides ApAs, AsAx, Ai Aa, respectively (not coinciding with any vertices). Show that the perpendicular bisectors of the three segments A,B; never concur. Solution Ay SOLUTIONS 3.5 ~ 3.7 45 Suppose they concur at P. Assume without loss of generality that PA, < PAz < PA3. Then Ai, Az lie in the closed disc with center P and radius PAs, hence Bs lies inside that disc, and we have PB3 < PA3. On the other hand, it has been assumed that P lies on the perpendicular bisector of ABs, and so PAs = PBs; a contradiction. 3.6 — Let aj, a2,a3,... be a sequence of real numbers satisfying the in- equality l@kam—@r—am|<1 forall k,m. Show that the following inequality holds for every natural numbers k and m: % am) Ld kom ie Solution ‘We first prove the auxiliary inequality laem — kam| < k. qa) We induct on &. For k = 1, (1) holds trivially. Assume (1) holds for a certain k (and all m). Then lagegtym = (# + 1am! S laemtm — Gm — Am|+ laim — kam| S14 |@em — kam) <1 +k, showing that (1) holds with & replaced by k + 1. So (1) is proved for all k and m. Of course, we may interchange k and m in (1): l2imtm — Gm — Om + im — kam| laem — maz| <_m. (2) From (1) and (2) we conclude [max — kam| < |@im — maz] + |aem — kam| 222, we have 21 > 22. Analogously, ©) >22>%3>°°:>En-1 Assume n > 7. Then tn-1 > 1, tn-2 > 2, ..+) ta-7 > 7, and hence Zn-1@n-2°*-Eq-7 > 7! > 1980, a contradiction, Son <7. On the other hand, for n = 7 there exist numbers 2; and a; as needed: (w1, 22, %3, 24,25, 26, 07) = (11,6,5,3,2,1,1), (a1, 42, a3, a4, a5, a) = (191, 336, 401, 663, 992, 1981). Thus n =7 is the greatest integer with the given property. 3.8 Let S be a set of 1980 points in the plane. Every two points of S are at least 1 apart. Prove that S contains a subset T’ of 220 points, every two at least V3 apart. Solution Take a line ! passing through a point P € S and such that the whole set S lies on one side of !. Let D be the closed half-disc limited by the semicircle centered at P, of radius V3, and by its diameter contained in line 1. Divide D into seven congruent sectors of angular size 1/7 each (x standing for the radian measure of 180°). We are going to show that each sector can contain at most one point of S, other than P. To prove this claim, take P to be the origin and line I to be the z-axis of a coordinate system. Remove from the middle sector of D all points which are within distance 1 from P. What remains, is a “curvilinear quadrilateral”, whose most distant points are cos$m,sin$n), B= (V3cos$m,V3sin $2), SOLUTIONS 3.8 ~ 3.9 AT with AB? = (/3 +1)” cos?($ nm) + (3-1) sin?(3 =) = 4-23 cost n <4—2V3 cosda It follows that there are no more than 8 points of S in D, including the center P. Delete all these points from S; what remains, is a set S’ of at least 1972 points. The same operation can now be performed with respect to S'. Continue this procedure until there are no points left. At each step we kill at most 8 points’ of S (having covered them with a half-disc of radius V3). Thus the number of steps is not less than 1980/8, hence not less than 248. The centers of the half-discs constructed in the successive steps constitute a set of at least 248 points, the mutual distance between any two of them exceeding V3. Remark At each step of the procedure we kill at most 8 points; true — but this number can only occur for some very specific configurations (within radius 3 from P). In “most” cases, such a half-dise D will cover 7 points or less (think what the situation looks like if we divide D into six sectors rather than seven). It seems that the number 220 from the statement of the problem, or the number 248 we have worked out, might be replaced by a significantly larger one. How large? In general, if we are given a natural number n, what is the greatest m (= m(n)) such that every plane set of n points, each two at least 1 apart, contains a subset of m points, each two at least /3 apart? What is the asymptotic relation between n and m(n), as n grows to infinity? What if V3 is replaced by another distance? The author does not know the answer to any one of these questions. 3.9 Through the opposite endpoints A and B of a diameter AB of a given circle, the parallel tangents | and m have been drawn. Let C be a point on J, other than A, and let qi, q2 be two rays drawn from C; assume that g; cuts the circle in points D;, Ei (D; lying between C and Hj); i = 1,2. Rays AD:, AD2, AEi, AE meet m in the respective points My, Mz, N, Ng. Prove that M1 M2 = NiN>. Solution Draw through C the tangent, other than I, touching the circle at P; line AP cuts m at Q. Clearly, points My, Mz lie on m on one side of Q and Nj, No lie on the other side. ‘The solution of the problem consists in noticing the following pairs of similar triangles: 48 SOLUTION 3.9 AAMQ~AAPD, AANQ~ AAPEL, (ay AAB\C~AD,AC, APEC ~ AD: PC. (2) The first relation in (1) follows from the angle equalities 4AM,Q = LAM, B = 90° - LBAM = 90° — LBAD, = LABD, = LAPD, (and ZQAM, = £PAD),); the second is analogous. Since CA is a tangent, we have ZAE,C = £D, AC, which (in view of LACE, = ZACD,) proves the first relation of (2); the second one results from the first upon replacing A by P (recall that CP is the other tangent) From (1) and (2) we obtain the proportions MQ_ AQ MQ_ AQ AE, _CE, PE, _ CE, RDM AD SRE Ai AD ye ACh Pre PC Hence _ PD: 4) PD: AB yp) ABy PDs MQ = Fp, 40= ap, PE, M12= Ap, PH, Me =Ch PC yon = Fe ng= ma, because AC = PC. Thus Q is the midpoint of MiNi. Subscript “I” may be replaced everywhere by “2”. This leads to the conclusion that @ is also the midpoint of MzN2. The final conclusion MMz = NyNz hence follows. SOLUTIONS 4.1 ~ 4.2 49 4.1 Find the least integer k for which the set {16,17,...,k — 1,k} contains 15 numbers 61,62,...,615 (pairwise distinct) such that bm is divisible by m for m = 1,2,..., 15 Solution ‘The numbers b2,b3,..., bis have to be composite. Since there are only 18 composites in the interval [16,33], we conclude that k > 34. And for k = 34 we can find the needed fifteen bjs in the set {16,17,...,34}. These by =34, 63 =33, bg = 16, by =21, bg = 32, by = 27, b= 22, bi= 24, bis = 26, big = 28, So the least k with the given property is k = 34. 4.2 The infinite sequence ao, a1,a2,... is defined by the recursion Qng1 = a2 +(an—1)? for n=0,1,2,... Find all rational numbers ap such that there exist four distinct indices k, m, p, q for which ay — ap = am — az. Solution Set 2a, — 1 = 2p, i.e. dn = (ta +1)/2. The given recursion formula yields Zng1 = 2an41 — 1 = 2a? + 2(an — 1)? —1 = 402 — 4a, +1 = 22. Hence by obvious induction ¢, = 23”. Write zo as an irreducible fraction zo = r/s (r,s integers, s > 1). ‘The postulated equality a, — ap = am — ay implies the analogous equality involving the 2,8; that is, ryt py? ppy2™ pp 2k @ 64 6 ©) Q Suppose for instance that q is the greatest index among k, m, p, q. Equality (1) multiplied by s* produces Pt m2? 28-2? 4,2 gam _ 2k gt —2k | with right side divisible by s (k, m, p, q being distinct by assumption). Since r/s is irreducible, this forces either r = 0 or s = 1. Assume r # 0 (and hence s = 1). Now suppose that e.g. & is the smallest number among k, m, p, ¢ and rewrite (1) as # m oP r 2 This is possible only for r = +1 (otherwise the right side expression is divisible by a strictly higher power of r than the left one). 50 SOLUTIONS 4.2 ~ 4.3 So we are led to the conclusion that zo is one of the three numbers 1, 0, 1. Evidently, for zo = r/s € {—1,0,1}, every quadruple of positive exponents k, m, p,q satisfies (1). ‘The corresponding admissible values of ao are: 0, 4, 1. 4.3 In triangle ABC, let r be the inradius, let r4 be the radius of the circle tangent to AB, AC and (externally) to the incircle of ABC; define rg, rc analogously. Prove that rattatrc2r, equality holding if and only if the triangle is equilateral. Solution Cc gf HA, T% T Denote the incenter by I and the center of the first small circle defined in the problem by I4. Suppose these two circles touch the side AB at T and T4. Let K be the foot of the perpendicular from I to IT. Considering triangle IK Ia: in(A) asin(2K Ign) = TE = PERK _ aa _ 1 (ra/*) sin($) =sin(2KI41) = iy 2 alle 6 en eG and hence ra _ L=sin(A/2) _ 1 cos((m—A)/2) _ van?(254 r 1+sin(A/2)~ 1+4cos((r—A)/2) 4 We thus have ra=rtan?a, rp=rtan?B, rc =rtan’y, where the angles a, 8,77 € (0, 7/2) are defined by notice that a+ 6+ 7 = 7/2. Writing u=tana, tang, w=tany=cot(a+f)= uto SOLUTIONS 4.3 - 4.4 51 Lew _ >uv+uw+ow =uvt(utv)- w Therefore ratte tre =r(u? +0? +w*) >r, equality holding only for u = ie, A= B=C=7/3. Remark We might also use the convexity argument and the Jensen inequality (see the Remark in the solution to problem 1.3). Consider the function f(z) = tan?e. It is strictly convex in the interval (0, 7/2), because its derivative f"(z) = 2(tan z)(1+ tan? 2) is strictly increasing in that interval. Consequently, ratrat+tTc ratte tre = w; equivalently, for a = 6 = 7 = 7/6, F(a) + £(8) + Fr) 29/(S2942) = a10"(Z) with equality only for a = 6 = y (= 7/6). 4.4 Suppose that n cells are arranged into a circle (n > 3). Each cell can be occupied by one of the two symbols 0,1. The following operation is admissible. Choose any cell C’ occupied by a 1, change it into a 0 and simultaneously change the symbols 2, y in the two cells adjacent to C'to their complements 1—2, 1—y. In the initial configuration there is a 1 in one cell and Os elsewhere. For what values of n is it possible to obtain 0s in all cells, in a finite number of admissible steps? Solution The answer to the question will depend on whether or not n is divisible by 3. If 3 divides n, color the cells successively white, red, blue, white, red, blue etc. Let sw, s-, 8 be the sums of numbers in white cells, red cells and blue cells, respectively. At each step of the procedure the change is performed on one cell of every color, so the sums sw, s-, 5» change their parities simultaneously. Hence, if we start e.g. from sy = 1, s, = 8 = 0, we will never atrive at sy = 8, = s4 = 0. If 3 does not divide n, it is possible to get zeros everywhere. We devise the following algorithm. Assume that at the start the only “one” is in the 52 SOLUTIONS 4.4 ~ 4.5 third cell from the left, relative to an appropriate cyclic enumeration of cells. Thus the initial configuration is 001000...0000. Consider the evolution: 001000...0000 010100...0000 011010...0000 ee eee O11111. 010 OL ior 1iliii1...1110 (at each step we choose the rightmost “one” and perform the allowed change, in that cell and in the two neighboring cells). In this way we obtain one zero and n — 1 ones. Now, if n = 3m-+1, then there are 3m ones. Divide them into blocks of three and perform m steps, in each step converting three successive ones to zeros; that clearly does the job. If n = 3m+2, we have 3m +1 ones in the configuration obtained (111111... 1110). Changing the contents of the last three cells we reach the state (111111... 1001), with two zeros and 3m ones, and we may repeat the method of the preceding case. Thus it is possible to achieve the configuration with all zeros if and only if n is not divisible by 3. 4.5 Let P(z) = 24 + a,2° + ajax? + age + a4 be a polynomial with rational coefficients. Show that if P(z) has exactly one real root €, then is a rational number. Solution It is evident that € must be a root of even multiplicity; otherwise P(x) would take negative values in a left or right vicinity of €, and this would force the existence of another real root (because lim P(z) = 00 as z —+ -too). Consequently € is also a root of the derivative P’(z), of odd multiplicity (1 or 3). Let D(x) be the greatest common divisor of P(x) and P’(z). The degree of D(z) is not greater than 3 (the degree of P’(z)) and not less than 1 (because D(é) = 0). The multiplicity of € as a root of D(z) equals its multiplicity as a root of P’(z) (either 1 or 3). Assume D(z) has degree 2. Every root of D(z) is also a root of P(z). So D(z) has no real roots other than €, and we obtain D(x) = (x — €)?; a contradiction, since € has to be a root of D(z) of multiplicity 1 or 3. SOLUTIONS 4.5 - 4.7 53 ‘Thus D(z) has degree 1 or 3. Writing P(z) = D(x)Q(z) we see that one of the factors D(z) or Q(z) has degree 1. In other words, either D(z) = e-€ or Q(z) = It remains just to notice that P’(x), D(z), Q(z) are polynomials with rational coefficients, this property being preserved under the division algorithm. So € is a rational number. 4.6 — The sequences of real numbers (tn), (Yn); (zn) are given by 1 1 1 Tn41= Unt Yn+1 Stato Zag. _ for n>0; Zo, Yo, 29 are given positive numbers. Prove that (tn), (Yn); (Zn) are unbounded sequences. Solution Let Sn = @n + Un +n. We claim that the sequence (sq) is not bounded above. Suppose it is. Then the sequence of reciprocals (1/sn) is bounded away from zero; say, 1/8, > ¢ > 0 for all n. Hence 1 1 1 Sn nti t Yost + 2nd = Bn tn tint OF + 1 1 trent trot; Sn on 1 > Int Yn tin + — Sn Sn obvious induction now gives sp > so + n- 3c showing that, after all, (sn) is not bounded above. This settles the claim. It follows that at least one of the three sequences (tn), (Yn), (zn) is not bounded above. This, in view of the recursive definition of these sequences, implies that each one of them is unbounded 4.7 Let a > 3 be an odd integer. Prove that for every positive integer n the number a?” — 1 has at least n + 1 distinct prime divisors. Solution Induction. Consider n = 1. One of the two integers u v = (a—1)/2 is odd, and both are greater than 1. Thus a? — an odd prime divisor, hence it has at least two distinct prime divisors (2 is the other one) Assume the claim is true for a certain n > 1. Consider n+1. Note that a") 1 = ay with 2 =a" $1, y= a" —1. Since a” = 1 (mod 4), ie., 2 = 2 (mod 4), x has an odd prime divisor p. By inductive assumption, y has at least n +1 distinct prime divisors; clearly, p is not one of them. So the product cy has at least n +2 distinct prime divisors; and this is just —~ what we need to complete induction. 54 SOLUTIONS 4.8 - 4.9 4.8 — The plane has been partitioned into N regions by three bunches of parallel lines. What is the least number of lines needed in order that N > 1981? Solution Suppose there are « parallel lines in the first bunch, y lines in the second and z lines in the third. We may assume that no three lines meet at one point. The x+y lines of the first and second bunch partition the plane into (x + 1)(y +1) regions. Let | be one of the remaining z lines. The z + y lines cut | into z+ y +1 pieces, and each one of these pieces splits one of the existing regions into two. So the number of regions is increased by a+ytl. ‘The same contribution comes from each one of the z parallel lines of the third bunch, Consequently, N=(e+l)ytl)+eetyt+l=ntme+l, where n=2+y+z, m=yz+2¢+zy. Note that ms (2? ty? +2) -3((y 2)? + (2-2) +(@-9)") Se pyr te? Hence wae? ty +2 4+2m> 3m, and so Nantmt+1entjn+l For n = 75 the expression on the right takes value 1951. Thus n = 75 is not enough. For n = 76 the right-side value exceeds 2000. If we try 6 = 2-+y+2, with the addends x = 26, y = z = 25, we obtain m = 26(25 + 25) +25 = 1925 and N = n-+m-+1 = 2002, which is enough. So N = 76 is the least number of lines needed. 4.9 Fora function f: [0,1] — [0,1] define the sequence of its iterates by fi(z) = f(z), frtt(2) = f(f"(w)) for 2 € [0,1], n=1,2,3,.... Given that |f"(z) — f*(y)| < |e — yl for a certain n and all a,y € [0,1], « # y, prove that there is a unique zo € (0, 1] such that f(x0) = 20. Solution Fix n. The given condition implies the continuity of f”. The difference 9(z) = f"(«)—2 satisfies g(0) > 0, 9(1) < 0, and hence by the intermediate value property there exists xo € (0, 1] with g(to) = 0, ie. f"(x0) = 20. SOLUTIONS 4.9 - 5.1 55 Consider 21 = f(2o). Clearly, £" (21) = F" (f(wo)) = F(F"(@0)) = F(vo) = 2 This forces 21 = x0; otherwise (assuming 21 # zo) we would obtain leo — ea] = [f"(20) — f*(21)| < leo - 2al, a contradiction. So (zo) = zo. If f had a fixed-point other than zo, it would be automatically a fixed- point of f*. The same argument as above shows that this is not possible. ‘Thus zo is the unique fixed-point of f. 5.1 Find all pairs of positive integers n,m with ged ((n +1)" —n,(n+1)"*9—n) > 1. Solution Let n,m be such a pair and let d > 1 be the greatest common divisor in question. Thus d divides the difference (n+1)™3 = (n+1)™ = (n+ 1)"((n +1)° - 1). Obviously d is coprime to n + 1, and so (n+1)8=1 (mod d). By the definition of d, (n+1)™=n (mod d). Consider three cases: Case (m = 3k). Then n=(nt1)™= ((n+1))'= (mod 4), ie, n = 1 (mod d), and thus 1=(n +1)? =(1+1)8 = 8 (mod d); hence d=T and n= 1 (mod 7) Case (m = 3k +1). Then = (n+1)™ =(n+1)((n+1)3)'=n+1 (mod 4), a contradiction. Case (m = 3k + 2). Then n= (nt 1) = (n+1)((n-+1)%)* =(n+1)?=n?42n+1 (mod 4), ie., n? +n+1=0 (mod d). This can be rewritten as n(n+1)=-1 (mod d). 56 SOLUTIONS 5.1 - 5.2 On the other hand, we have (in view of n = (n+ 1)?) n(nt1) = (n+1P(nt+ 1) = (n+ 8 = (mod d). Hence 1 = —1 (mod d), yielding d = 2; a contradiction, because n(n + 1) is even. Summarizing, (m = 3k) is the only case possible. The conclusion in that case was n=1 (mod 7), m=0 (mod 3), and it is easy to verify that every pair n,m satisfying this last system satisfies also the condition of the problem (each one of the two numbers (n+1)™—n and (n+ 1)™+5—n is divisible by 7). Thus the general solution of the problem is constituted by all pairs (n, m) = (7! — 6,3k) where / and & are any positive integers. 5.2 Let C be a circle with center O and radius 1, and let F be a closed convex region inside C. (This means: if P,Q are points of F then the segment PQ is contained in F; all boundary points of F belong to F; all points of F belong to the open disc limited by C.) Furthermore, assume that from each point of C we can draw two rays tangent to F and making an angle of 60°. Prove that F is the disc centered at O, with radius 1/2. Solution Choose a point P on C and draw the two rays from P, touching F. They cut the circle in points Q, R, and we have LQPR = 60°. Now draw from Q the other tangent to F, cutting C in R’, and draw from R the other tangent to F, cutting C in Q’. Each one of the angles ZPQR! and LPRQ' has size 60°, according to assumption. So the lines QR’ and RQ! are parallel, and both are tangent to F. Since F is convex (and not contained between these lines), they must coincide. Hence Q = Q’, R = R’. All angles of triangle PQR have size 60°; the region F is contained in each one of the wedges PQR, QRP, RPQ. In other words, F is contained in the equilateral triangle inscribed in C, having one vertex at P. P SOLUTIONS 5.2 ~ 5.3 57 Since P could be any point on C, we conclude that F is contained in every equilateral triangle inscribed in C. Denote by D the intersection of all those triangles; D is the disc with center O and radius 1/2. We have shown that F C D; it remains to prove the opposite inclusion. Let X be a point on the circumference of D. Assume X ¢ F. Let I be the line through X, tangent to D. Since F is contained in D and does not contain the point X, line ! is disjoint from F. Move | by parallel translation toward the center of C until it touches F, and denote the resulting position of that line by I’. Let l’ intersect C in P and Q. Draw from P and Q rays p and q, other than I’, touching D. Each of these rays makes with /’ an angle smaller than 60°, and the region F is contained in the triangle formed by pieces of p, q and I. This contradicts the condition of the problem. Consequently, every point on the circumference of D belongs to F. Hence, by convexity, the whole disc D is contained in F. The two inclusions F C D and D C F result in the set equality F = D, which is exactly the assertion of the problem 5.3 Letn >2bean mee Prove the equality Tla(Z0+52 #5))= T(§ (1- -s)): Solution Write: t, = tan ——— watn(E(1+525)), ne atn(E(1- 52). In view of tan3a = tana(3—tan?a)/(1—3tan?a) we have Bhd ta = tan(3. ea ) and in view of tan(a + #) = (tana + tan#)/(1Ftanatanf) we have a 3tle V3+te eee n=tn(7- 22) 2 Ving FSOONG 3-1) 14-8 So upve = teyi/te and ® 4 _ tz ts. ther _ tran _ ten(+ gq) [1 (usm) = 2-2-8 = eH = i tr th th tan and the claim follows. 58 SOLUTIONS 5.4 ~ 5.5 5.4 For a positive integer x let P(x) denote the product of all digits of x (in decimal system). Let 2; be any positive integer. Consider the sequence (2n) defined recursively by tng = tn+P(zq). Prove or disprove that (z,) is necessarily a bounded sequence. Solution We claim that (zq) is necessarily bounded. Suppose it is not. Let m be an integer such that 9™+! < 10™ and x; < 1041. Let k be the least integer such that 244; > 107+? (it exists in view of our assumption that the sequence is unbounded). Thus 2, < 10™+!. Consider the decimal representation of zy: Bk = 10m +10" mar +--+ 10? c2 + 10c1 + 0 Clearly, P(zz) = coci -++¢m < 974 < 10” and hence Tey = Te + P(e) < 10+? + 10". Since z441 > 10+, we see that 2441 has a leading 1 followed by a 0. Therefore P(ze41) = 0 and so Teo = Ter + P(tey1) = tey, which means that the sequence (zj) is constant from n = k+1on, contrary to the assumption of unboundedness. Contradiction proves the claim. 5.5 Suppose that the closed interval [0, 1] has been partitioned into two disjoint subsets A, B: AUB=(0,1], AnB=6. Show that there exists no real number a such that B = A+a, where by definition A+a={y: y=a+a, 2€ A}. Solution Assume there exists a such that B = A + a; without loss of generality let a > 0 (in case a < 0, replace a by —a and interchange the roles of A and B). Since B is contained in [a, 1], the interval [0,a) must be contained in A. Consequently [a, 2a) is contained in B, hence it is disjoint from A. So the interval [2a, 3a) is disjoint from B. Suppose 3a > 1. Then (2a, 1] is contained in A; in particular, 1 € A, implying 1+ a € B, which is not possible (the set B cannot stick outside the interval [0,1]). Therefore 3a < 1 and hence the interval [2a, 3a) is contained in A. Repeating the reasoning we show that the interval [4a, 5a) is a subset of A, 80 is (6a, 7a), so is [8a, 9a) etc. This process never stops. Thus 2na € A for every natural n; and this is absurd because A is a subset of (0, 1] and a is a positive number. Contradiction ends the proof. SOLUTION 5.6 59 5.6 Let a be a given integer. Find all real-valued functions f(z) defined on integers 2 > a, satisfying the functional equation f(e+y) = F(a) f(y) for all z,y such that 2 >a,y>a,2+y>a. Solution Case 1. a> 0. Consider three subcases. Case 1.1. f(a) = 0. We claim that then f(e)=0 forall e>a, (a) proving this by induction. Pick ¢ >a and assume that f vanishes at ¢—1. Clearly, 2z > a, so f(2z) is defined and we get 0= f(e—1)f(e@ +1) = f(e@-1 4241) = f(22) = fle +2) = f(z)’. Thus f vanishes at 2; claim (1) is proved. Case 1.2. f(a) #0, f(a + 1) = 0. We repeat the above proof to show f(2)=0 forall 2>a41. Since f(2a) = f(a)? #0, 2a cannot exceed a. Consequently a = 0. And since 0% f(0) = f(0-+0) = #(0)?, we get f(0) = 1. Conclusion: _ fi for2=0, a= {5 fore >. (2) Case 1.3. f(a) #0, f(a +1) £0. Denote the ratio f(a + 1)/F(a) by 4. We assert F(z) = flag?-* forall 2 >a. This is true for « = a, r= a+1. Fix x >a and assume inductively that the inequality holds for « — 1 and 2: f(e@-=F@ge*, fle) = Fla)g?-*. Again we have 2x > a, so f(2z) is defined and can be evaluated in two ways: f(2z) = f(e-1 +241) = f(@— Ise +1) = f@a**F(@ + D), f(2x) = fe +2) = f(2? = f@)?"™*. Equating these expressions we conclude (in view of g # 0, f(a) # 0): f(x +1) = f(a)g?*'~¢, proving the asserted equality for z + 1, and, by induction, for all 2 > a. Setting in that equality 2 = 2a we obtain f(2a) = f(a)q*. On the other hand, f(2a) = f(a)?, Hence f(a) = q* and consequently f(2)=9" forall z>a. (3) 60 SOLUTIONS 5.6 - 5.7 Case 2. a <0. The function f restricted to nonnegative integers satisfies the conditions of case 1 (with 0 in place of a). Hence f is given by one of the three formulas (1), (2), (3) for x > 0. (In the considerations that follow it is essential that the number —1 is in the domain of f.) Case 2.1. f(x) = 0 for x > 0 (formula (1)). Then also f(z) = 0 for a 0 (formula (2). Pe case is impossible because f(0) = f(1 — 1) = f(1)f(-1) =0- f(-1) = Case 2.3. f(x) = g? for z > 0, for a certain g # 0 Coan (3)). We assert that then fle) g fora < « < 0, too. First, note that 1= 9° = f@) = f(1- 1) = fQ)F(-1) = a F(-1), and 80 f(-1) = 7. Again induct backward. Assume f(z) = q® for some z with a < z <0. Then f(x — 1) = f(x)f(-1) = q?q7! = q?7}, and the assertion follows. Conclusion. Regardless of the sign of a, the functions (1) and (3) (with any exponentiation base g # 0) constitute the general solution of the given functional equation, except for a = 0, in which case there is one more solution, given by formula (2). 5.7 Find the triple of positive integers (x,y,z), with z least possible, for which there exist positive integers a, 6, c, d with the following properties: (i) a@act, r>a>c; (i) z=ab=ed; (it) e+y=atb. Solution Suppose z, y, z, a, 6, c,d satisfy the given conditions. Since a! the numbers a and c have exactly the same prime factors: a=[Je, c=] x, iat and we have ajb = 7d for i = 1,...,n. The conditions a > ¢ and ab = ed imply 6 < d; hence a; > 7 and so we have, for each i, . =F =f=[]a. ist oe % Denote this ratio by q; it is an integer greater than 1. Thus a = gc, d= qb and from a’ = c4 we obtain the equation qz=et}. SOLUTIONS 5.7 - 5.8 61 It is an easy exercise to show that c = q = 2 is the unique solution of this equation in integers greater than 1. So we get a= 4, d = 2b, z = 4b. Now we can reformulate the problem as follows: Find the triple of positive integers (x,y, 6), with 6 least possible, such that a =2, g>4, ety=44o. ‘The first two conditions force x = 2, k > 2, and we arrive at the system of equations ky=2b, M*+y=446, with unknowns k > 3, 6 > 1, y > 1 (integers); we are looking for the solution in which 6 is a minimum. ‘Try k = 3. The system becomes 3y = 2b, 8+y = 4+6, with the unique solution 6 = 12, y=8 Now assume k > 4, Then 4+6 > 24+y > 17, whence b > 13. So the minimum 6 is = 12. The corresponding values of k, 2, y and z arek =3,2= 2 =8,y=8, z= 4b = 48. ‘The triple (z, y, z) = (8,8, 48) is the solution of the problem. 5.8 Let ABCD be the regular tetrahedron with edge length 1 and let P be a point inside ABCD. Show that d(P, AB) + d(P, AC) + d(P, AD) + d(P, BC) +4(P, BD) +d(P,CD) > v2, with equality only for P being the centroid of ABCD ; the symbol d(P, AB) denotes the distance from P to line AB. Solution Consider the cube of edge length $2 and choose four of its vertices spanning tetrahedron ABCD of edge length 1; the edges of the tetrahedron are the diagonals of four faces of the cube. The distance from P to the diagonal of a face is not smaller than the distance from P to the face. Since the six distances from P to the six faces of the cube sum up to the value 22, the claimed inequality follows. Equality occurs if and only if all the six estimates (used above) turn into equalities. The numbers d(P, AB) and d(P, CD) are equal to the distances from P to the respective (parallel) faces of the cube if and only if P lies on each one of the two planes perpendicular to those faces and passing through line AB or CD, respectively. The same argument applies to the two other pairs of parallel faces of the cube. The unique point that the respective planes have in common is the centroid of the cube, which coincides with the centroid of the tetrahedron. 62 SOLUTION 5.9 5.9 Let S.= —=—== . Find a positive constant C such that x = VP +R the inequality n 3. Note. The smaller C, the better the solution. Solution ‘The left inequality does not involve the constant C and can be proved by induction. For n = 3 it is verified directly. Assume inductively that Sp-1 > n—1 holds for a certain n > 4; we must prove S, > n. Note that i Sn-Sn-i= ) aes jyksmaxG,ejen VPP EP eee ; La Dlr tae Fa VP +n G+ 2n' 1 1 1 > Da Daa + Tat at for nba ~ on™ So S, >n; induction is completed. For the right inequality, we can use j? +k? > 2jk to show that where Since Vk - Ve—1 , we obtain by “telescoping” Ve iS 1 1 Vet Ve=1 > VE Tp < 230(VE~ VET) = 2va. tat Hence Sp < (1/V2)(2Vn) = 2V2n, and we are done; one can set C=2w. SOLUTIONS 5.9 ~ 6.2 63 Remark The constant C = 2v/7 is not the least possible. We have = ~S-D gest Lae i 1 Laer? Loe < of Foes = Bint + v2) (just observe that the sum in the middle line is a lower approximation to the integral; to evaluate the integral use substitution y = r+V1 + 2? ). Setting So = 0 we obtain (again by “telescoping”) Sn < Cn with C = 2In(1+¥2). This constant is the best possible because the approximation of the integral by sums becomes sharp as n grows to infinity; we leave the details as an exercise in undergraduate calculus. 6.1 Nonnegative real numbers a, 6, 2, y satisfy a°+65 <1, 25 +y5 <1. Show that a?x3 + 6?y3 <1 Solution Apply the arithmetic mean-geometric mean inequality to the following five numbers a8, a, 2°, 25, 25: 23 = Yababe5a825 < 1 (a5 + a5 +25 425 +25) Likewise, we have By? < 2° 4 B98 Hence a?29 + by? < 3(a° +65) + F(a +98)

1 an integer, satisfying Pp +1) +494 1)=n(n +1). Solution Rewrite the equation as P(p+1) = (n-a)(n+9+1). (1) Bither n—g or n+q-+1 must be divisible by the prime p. The first possibility would imply p 2. Equation (1) after reduction by p becomes p +1 = (kp — 2q— 1)k, i., 2kq = (k + 1)(kp— p- 1) (2) The prime g must divide one of the factors on the right. Assuming that q divides kp — p — 1, so that kp— p—1= ql, we get from (2) 2k = (k+ 1), which is impossible for k > 2. Therefore q is a divisor of k +1. Writing k-+1 =m (m> 1 an integer) we obtain from (2) 2k = m(kp — p— 1). (3) Obviously, k is coprime to k +1. Thus & divides the second factor on the right side of (2): kp—p—1 = kr (r an integer); so (3) becomes 2 = mr, which means that m is equal either to 1 or to 2. Accordingly, equation (3) yields either k = (p+ 1)/(p—2) or k = (p+1)/(p—1). The ratio has to be an integer. Hence: In the first case, p = 3 or p = 5. In the second case, p = 2 or p= 3. So there are four subcases possible: Subcase (m = 1, p = 3). Then k = (p+1)/(p—2) = 4, 9 = (k+1)/m and n= kp—q-1=6. Subcase (m = 1, p=5). Then k= 2,q9=3andn=6. ). Then k = (p+1)/(p-1) = 3, = (k+1)/m = 2 5 Subcase (m = 2, p andn=kp—q-1 Subcase (m = 2, p= 3). Then k = 2, q = (k+ 1)/m = 3/2 is not an integer. Thus the general solution is constituted by the following triples (p, 9, n) (3, 5,6), (5, 3,6), (2, 2,3). 6.3 A bounded planar region of area S is covered by a finite family F of closed discs. Prove that F contains a subfamily consisting of pairwise disjoint discs, of joint area not less than S/9. Solution We induct on n, the number of discs in F. For n = 1 there is nothing to prove. Fix n > 2 and assume the statement is true for every bounded planar region covered by a family of less than n discs. Let now F be a family of exactly n discs, covering a region R of area S. Let D be the disc in F of maximum radius r, Assume that D has area less than S/9 (if not, we are done). Then the disc D’ concentric with D, of radius 3r, has area less than S; hence there are points of R outside D’. SOLUTIONS 6.3 ~ 6.5 65 Partition the family F into two nonempty subfamilies F;, Fo, letting F, consist of those discs that have points in common with D and Fy consist of those that do not have (the disc D itself is in F;). Let R; be the union of discs in subfamily F; and let 5; be the area of R; (i = 1,2). Discs that belong to F; are contained in D’, by the maximality of r. Thus Ri C D! and Sy < 5! = area of D’; i.e., $1/9 < area of D. By inductive hypothesis one can select a number of pairwise disjoint discs of family Fo, of joint area not smaller than 52/9. Adjoining to Fo the disc D, we obtain a set of pairwise disjoint discs whose areas sum to a value T'such that T > Sp/9+ (area of D) > S2/9+ 1/9 > S/9, just as needed. This completes induction 6.4 The set N of positive integers has been partitioned into two disjoint sets A, B, their union exhausting all of N. Show that for every n € N there exist distinct integers a,b > n such that the three- element set {a,b,a +5} is contained either in A or in B. Solution At least one of the sets A, B — say, A — is infinite. Fix n € N. We can find in A three numbers ¢, y, z with z>n, y> 2x, z>2y. The six numbers 2, y, 2, y—z, 7—Yy, 2-2 exceed n and are pairwise distinct; this follows from 2 = 22-2 0 for §=1,2,3,4. Let u; =ai/2, v; = Yai, i= 1,2,3,4. The expressions in (1) (case n = 4) become 2 2 (ur + wave + wavs + ugva)” = (aii + aze2 + age3 +0424) = (uP + uh + uf + uf) (v2 + 0} + 03 + v2) = (ajay + ada, + a3r3 + a3x4)(r1 +22 +23+24) = 07, so that (1) turns into equality. Since the case v; = v2 = v3 = v4 = 0 (i.e., 21 = 22 = £3 = 24 = 0) must be excluded, we have aJe=rJ/e for i= 1,2,3,4, for a certain real constant A. Thus a; = ) for all i such that 2; #0. But a1, 2, ag, aq are all distinct, so three of the 2;s must be zero. According to the first equation, the remaining 2; is 1, and then the second equation yields that c equals the corresponding aj. In conclusion, the given system has the desired property if and only if c is equal to one of the four given numbers a1, a2, a3, a4. Note also that the a;s need not be positive. Remark We give a short proof of (1), implying also the condition for equality. If all vjs are zero, there is nothing to prove. If this is not the case, write SHU +--+ +Uprn, usupte tu, vate deb. Then ted if = uv? — 267v + s?v = v(uv ~ 8). os Dou = su)? = 0 ya - 29> i= i= Since v > 0, the claimed inequality s? < uv follows. Equality requires that 0, ie, weedy for i vu; — 504 L...yn, where A=s/v. 6.6 — There are given six straight lines in the space. In every triple of those lines, two lines are perpendicular. Show that the given lines can be labeled Jy,...,/6 in such a way that ty, lo,l3 are pairwise perpendicular, and so are l4, ls, le. Solution Choose one of the given lines ! and consider the following two (comple- mentary) cases: SOLUTIONS 6.6 ~ 6.7 67 (i) There are three other lines !’, !”, !”” perpendicular to !. (i) There are three other lines 1", !”, !’" nonperpendicular to l. In case (i), the triple (W', ",’") contains a pair of perpendicular lines, say, l'L1", Thus the lines J, !, !” are mutually perpendicular. In case (ii), each one of the triples (1,1’,”), (1,1",0"), (1%, 0") contains a pair of perpendicular lines. Since / is not perpendicular to any one of V1", 0”, it follows that U, 1”, l’” must be mutually perpendicular. In each case we have found a triple of pairwise orthogonal lines. Denote them by hy, la, ts, and let ly, Is, ls be the remaining lines. We have to show that IgLisLisLla. Assume that e.g. [, and ls are not perpendicular. Each one of the triples (11, la, ls); (lo, lays), (ls, 445) contains a pair of perpendicular lines. Thus {, is perpendicular to either l4 or {s, and the same can be said of ly and of I. Consequently, either ls or Js is orthogonal to some two lines of Ly, laylg. Lete.g. th, lal ly. Since also Ig lh, Ig. ly, it follows that ls[|l4. Hence the triple (Is, 4,5) contains no orthogonal pair; a contradiction. ‘Thus 41 Js, and similarly 141 le, Is-L le. 6.7 Let Pi, Pz, Ps, Pj be four distinct points in the plane. Suppose [1, In, Ig, Ig, Is, Ig are segments in the same plane, with the following property: every straight line passing through at least one of the points P; meets the union I; U Ip U Ig U 14 U Ig U Ie in exactly two points (we agree that the endpoints of each I; belong to J;). Prove or disprove that the segments I; necessarily form a hexagon. Solution ‘The following example disproves the statement. 68 SOLUTIONS 6.7 - 6.8 In coordinates, let Py = (2,0), Pe EC rC—si—Csr—_—CisCSCizsKCzarsSN and look eo ce disjoint triangles ABC and DEF. Take for hh,..., Ie the segments AB, BC, CA, DE, EF, FD. They do not form a hexagon, yet they satisfy the condition of the problem, 6.8 (a) Prove that (2"+!—1)! is divisible by [[(2"**~! — 1)”, for every natural number n. io (b) Define the sequence (cn) by c1 = 1, ¢n = ((4n — 6)/n)cn—1 for n > 2. Show that each cp is an integer. Solution : (a) Write Np =(2"t2=1)!, Dn = PTi(2"t!-#- 1)", Qn = Nn/Dn- Thus Do = 1, D: =3, Dz = 3-7, D3 = 34-77-15, Dg = 38-74-15?-31, and in general, Dn = D,2,(2"+1 — 1). Hence Qn _ NyDn21 _ (art? — 1)! _ ee - ) Qn Nn? (@ =)" = 1) ar is an integer for each n > 1. Since Qo = No/Do = 1, all Qas are integers. (b) As in part (a), binomial coefficients are the clue. The initial cps are: > e=e=1, c=2, 4 =5, cs=14, c6=42, Have you ever met these numbers? Look closely into the Pascal’s triangle and try to see them: 6 dt, 21 | 35) 30) 7 1 de 5) 282 062 70 067 287 8) 1 9 36 84 126 126 84 36 9 1 1 10 45 120 210 252 210 120 45 10 1 ce ne os 42 = 252 — 210, and so on. In general, we guess that fi fan Qn ea =(T)- (0 for all n > 1. Once guessed, this is easily proved by induction, using the recurrence formula that defines the sequence. And since the binomial coefficients are integers, the claim follows. SOLUTIONS 6.8 - 6.9 69 Remark The cps of part (b) are called Catalan’s numbers. A more standard formula to define them is oe = 2” eel (but it does not so readily imply that they are integers). The Catalan numbers occur in many problems of combinatorics. For instance, ¢n41 is equal to the number of paths from (0,0) to (n, n) consisting of n unit steps north and n unit steps east, passing only through lattice points (2, y) with 2 > y. 6.9 To each side of the regular p-gon of side length 1 there is attached a 1x k rectangle, partitioned into k unit cells; k and p are given positive integers, p and odd prime. Let P be the resulting noncon- vex star-like polygonal figure, divided into kp +1 regions (kp unit cells and the p-gon). Each region has to be colored in one of three colors, adjacent regions having different colors. Furthermore, it is required that the colored figure should not have a symmetry axis. In how many ways can that be done? Solution = n Fix the color of the p-gon. Each one of the p cells adjacent to the p-gon can be colored in any one of the two remaining colors; the same regards the next bordering cells, and so on. We arrive at the number of 2? colorings, given the color of the p-gon, disregarding the condition of nonsymmetry. ‘The polygon P has p axes of symmetry. Let us fix one of them, call it 1, and count the number of colorings which are symmetric with respect to I. Once we color the cells of the rectangle bisected by / and the (p—1)/2 rectangles lying on one side of /, the coloring of the remaining rectangles is determined. This can be done in 2*°+1)/? ways. We have to multiply this outcome by p (the number of symmetry axes) and subtract repetitions. 70 SOLUTIONS 6.9 ~ 7.1 Note, however, that if a certain configuration is symmetric with respect to two distinct lines, then it is invariant under rotation by some angle 2j7/p, 0 f(n) such that the greatest common divisor of n and f(n) is a maximum Solution Assume that the decimal representation of n uses digits « and y (x # ). Denote the greatest common divisor of n and f(n) by g(n). Then g(n) is also a divisor of n + f(n) = 1111-(e¢+y) =101-11-(2+y). ‘A number of type (eeey)io or (ezyz)10 or (zyez)10 or (zyyy)io or (ezyy)io or (2yye)io is not divisible by 101 (easy exercise). Thus if n is of one of these types then g(n) necessarily divides 11-(z+y) and hence does not exceed 11-17 = 187. Now let us examine the remaining type n = (zyzry):0. Let (zy)10 = a, (ye)io = 6, ged (a,b) =d. So n=101-a, f(n) = 101-6, g(n)=101-d. To maximize g(n), we have to maximize d. From n > f(n) we obtain z > y. Surely d must divide a— 6 = 9(2 — y). If 3 does not divide d, then d has to divide 2 — y; hence d < 9— 1 =8 in that case. Suppose 3 divides d. Then of course 3 divides « + y, so that a = (zy)i0 is one of the numbers 21, 51, 42, 81, 72, 63, 54, 93, 84, 75, 96, 87. The greatest common divisor d of a and 6 = (yz)1o equals accordingly 3, 3, 6, 9, 9, 9, 9, 3, 12, 3, 3, 3; the greatest value d = 12 occurs when a = 84, b= 48. This gives n = 8484, with f(n) = 4848 and g(n) = 101-12 = 1212, the maximum value available for g(n). 7.3 Let n > 2 be an integer and let a, 21,...,2, be positive real numbers. Prove that £ = Gaeaea a+r. 22423 and find conditions for equality. Solution Using (twice) the inequality between the arithmetic mean and the geo- metric mean we obtain (writing tn41 = 71) ins (SS 2 (2D +2) = 72 SOLUTIONS 7.3 — 7.4 This is exactly the inequality we wish to prove. Equality requires that each time the means inequality is being used, the averaged terms should be equal. This yields the system of equations ati-®2 tees aen-1 = , qa +22 t2+23 rte ay t22=a2+23= tm ta. (2) If n is odd, equations (2) imply 21 = 22 =- (1) are also satisfied in that case. If n is even, equations (2) force zn; Clearly, equations @pSagS Seq Hie, 25 a45- and then, by (1), For a = 1 the last equation is automatic. When a # 1, we obtain a = f. The conclusion follows: the given inequality turns into equality if either all 2js are equal or, in the case of a = 1, n even, when all even-indexed as are equal and all odd-indexed 2;s are equal. 74 Ay A2AsAqAsAoAr is the regular heptagon and P is a point of its circumcircle, lying on the minor arc A7A;. Show that PA + PAs + PAs + PAr = PAz + PAa + PAs. Solution This standard equality can be justified without much trouble by simple manipulation on trigonometric expressions. A shorter proof is also available: Ag As As SOLUTIONS 17.4 ~ 7.5 73 Let PA; = 2; for i=1,...,7. Denote the length of the side and the length of the shortest. diagonal of the heptagon by a and 6, respectively: AjAg = ApAg == ArAy =a, Ay A3 = ApAg = ++: = A7An = 6. Applying Ptolemy’s theorem to each of the quadrilaterals PAs—1A:Ai41 (where, cyclically, Ao = Ar, As = Ax) we arrive at the system of equalities = 2,3,4,5,6, br; =az,—-azz7, bry = are —ary. be; =azi-y+azi41 for Thus bay + bas + bas + bez = axe + 2axq + 2azg — az, — azz, bare + bag + beg = az; + 2ars + 2azs + azz. Subtracting these two equalities we obtain (2a-+ 6)(x, +23 + 25+ £7) = (2a+6)(r2 +24 +26), whence zy +egtas+ a7 = 22+ 24426. Obviously, the same argument works for every odd n-gon. 7.5 Let n > 3 be an integer and let a1,...,an, be given nonnegative integers, pairwise distinct. Find all (n + 1)-tuples of nonnega- tive integers (21,...,¢n,), with the greatest common divisor of 21,..+)2n equal to 1, such that 4), + 42%2+-+-++ Anta yr, at) + a3t2+++++ aitq Solution Let 21,...,2n,y satisfy the required conditions. Adding the equations of the system we obtain (a1 +++ + an)(ti +++ tn) = y(ei +--+ 20). The sum 21 +-++-+ tn is positive (otherwise all 2; would have to be zero, contrary to the condition that ged(21,...,2,) = 1). Thus y=arte-tan (a) Rewrite the given system of equations in the form Voatigie = ye, for k=1,...,n; a 74 SOLUTIONS 7.5 ~ 7.6 of course the index i+1—k is to be understood (mod n). In view of (1), we conclude that Soa(eigi-e-2e)=0 for R= 1.0.0. (2) i Let zp = min{z1,...,2n}, ty = max{z1,...,2n}. Set in (2), first, k=p, and then k= q: Voaleisi-p— 2) =0, Yo ai(tiging — 2; isi i=1 In the first sum, all summands are nonnegative; in the second one, all summands are nonpositive. Thus all those summands are zero: ai(zigi-p—2p)=0 for #=1,...,0, (3) (4) Since n > 3, we can choose r € {1,...,n}, different from p and q. Setting in (3) i= p+r—1, and in (4) i=g+r—1, we obtain ai(tixi-g—2)=0 for i= Qpyr—1(@r— 2p) =0 and dg 4p—1(Bp — 22) = 0; and setting either in (3) or in (4) i= p+q—1: p4q—1(q — Zp) = ‘The numbers ap4r—1, Gg4r-1, Ap4q-1 are pairwise distinct (by assump- tion). Thus at most one of them can be zero, and therefore at least two of the three differences 2, — 2p, t+-q, t—2p must be equal to zero. Conse- quently, zp = 2, = zy. By the definition of zp and ¢,, this means that all zis are equal, their common value being 1 (because ged(z1,...,2n) = 1). So (21,..y2n)¥) = (1,-0.1, a1 -+-++-a@q) is the unique solution. 7.6 Ina folk dance, the dancers are standing in two rows, n boys facing n girls. Each dancer gives her/his left hand to the person opposite or to her/his left neighbor or to the person opposite the left neigh- bor. The analogous rule applies to right hands. Nobody gives both hands to the same person. Find the number of possible configura- tions. Solution Call the boys, from left to right, Alan, Bill, Charlie, Donald, ...; sup- pose they are facing Ann, Betty, Claudia, Doris, .... Alan joins his left hand with Ann’s right hand. We shall say that Alan’s right hand, Ann’s left hand, Betty’s right hand and Bill’s left hand form a “box” AB. Box BC is defined similarly, with reference to Bill, Betty, Claudia, Charlie; and so on. In each box we have three configurations available. Encode them SOLUTION 7.6 75 by symbols 0, 1, 2 as follows. In box BC: if Bill joins his right hand with Betty’s left hand and Charlie joins his left hand with Claudia’s right hand, we have configuration 0; if the two boys join their hands and the two girls join their hands, this is configuration 1; the remaining “diagonal” configuration is given code 2. In box AB, state 0 is not possible (Alan should not give both his hands to Ann); similarly, state 0 is not admitted in the rightmost box. State 0 in box BC excludes state 0 in box CD (Charlie should not give both his hands to Claudia); the conditions of the problem involve no other restriction concerning relations between these two “boxes”. As there are n — 1 boxes, the admissible configurations are represented by (n — 1)- tuples (“strings”) of entries 0, 1, 2 without any two Os in succession, entry 0 excluded on the outer positions. Denote the number of such strings by up. Obviously u; = 0, u2 = 2. Now consider a string of length n. It could have arisen from any (n—1)- string by attaching either 1 or 2 as the n-th entry; it could have also arisen from any (n — 2)-string by attaching 01 or 02 on the last two positions. The number of admissible (n — 1)-strings equals un; the number of admissible (n — 2)-strings equals u,—;. From each string of any one of these two types, we could have produced a “good” n-string in two ways, by choosing one of the actions described in the last paragraph. All this taken into account, we arrive at the recursive equation Unt = 2tn + 2ttn—1- qa There is a well-known method to handle equations of this type. One postulates tin = an" + by", (2) where (2) and (y") are geometric progressions satisfying the same recur- sion: 2"+1 = 22"422"-!) y+! = oy" 49y"-1, Thus x and y are the roots of the characteristic equation: )? = 21+2; ie, 2=1+¥3,y=1- V3, and (2) becomes Un = a(1 + V3)" +b(1- v3)". Setting n = 1,2 (and knowing the initial data u, = 0, u2 = 2) we obtain a system of two linear equations with unknowns a and 6, from which we determine a = 4(3- ¥3), 6= 4(3 + V3). Thus, finally, 3(3 - v3) (1+ v3)" + (3+ V3) (1 - v3)" BV ((1+- v3)" ~ (1 v8)""). Of course, one can verify directly that the last formula defines a (unique) sequence satisfying recursion (1) together with the initial conditions u; = 0, ug = 2. Un " 76 SOLUTION 7.7 7.7 We are given a rectangular array ay Ain Gm +++ mn of real numbers with |a;;| < 1, such that each column sum aij +-+++ am; equals 0. Prove that by rearranging the elements in each column one can obtain an array bu bin bat ses Bonn such that each row sum satisfies |bj) +--+ + bin| <2. Solution Such rectangular arrays are usually called matrices Let A= (aij) cm, jcn be the given matrix and let B= (bis) cm be the matrix whose each column is a permutation of the corresponding column of A and such that, denoting the new row sums by bu ++ bin = 81, the sum |s;|+---+|Sm| is a minimum. (There are only finitely many rearrangements, so the minimum value is attained.) We will show that the required condition |s;| < 2, ..., |sm| <2 is then satisfied. Assume this is not the case and |s,| > 2 for a certain k; there is no loss of generality in assuming just s; > 2, as the conditions of the problem are not affected upon replacing each aj; by —aij. Each column sum (in A, hence also in B) is zero, and therefore $1 +-+-+8m = 0. So there exists an s; < 0. In view of the inequali- ties Se = be +--+ +bin 2>2>0, 81= br t---tbn <0 we have bi- > bir for some r. Write d = bk, — bir. Since all entries are not greater than 1 in absolute value, 0 2>d > 0, we see that |s;| = |sz| — d; and since s; <0 |s;|. Contradiction ends the proof. Remark The constant 2 is the best possible. (For specific values of m an n one can, in general, work out a better bound for the absolute values of row sums available; think of the trivial case m = n = 1 or, slightly less trivial, m =n =2). Yet, the number 2 is the least constant that works for every m and n. In fact, it is the least constant that works for n = 2 and an arbitrary m. To see this, take an odd m and consider the two-column matrix, each column consisting of (m+1)/2 entries equal to (m—1)/(m+1) and (m—1)/2 entries equal to —1. Any rearrangement of terms in columns will produce a matrix with two positive terms in at least one row. The corresponding row sum will be 2(m— 1)/(m-+ 1), approaching the bound 2 as m becomes large. 7.8 Consider the functions fole)=22, fle) = =a for z>1. Prove that for every real numbers a, b with 1 1 and a k-tuple of indices i, i,...,é € {0,1} such that a < fag (Fy y(---Uin())--)) 1 and assume the claim is true for q — 1; 78 SOLUTIONS 7.8 — 7.9 that is to say, every dyadic fraction with denominator 2!~? can be obtained in the fashion as described. Take a v = p/2° € (0,1). Either v or 1—v equals half a number u € (0, 1) which is a dyadic fraction with denominator 29-1, Accordingly, either v = go(u) or v = gi(go(u)). Since by hypothesis u can be represented as a finite composition of g:s applied to the number 1/2, the same is true of v, and the assertion for q results; the lemma is proved. Returning to the problem in the original setting, choose an interval (a, ) with 1 R, other than f(z) = 1 and f(x) = z +1, satisfying that equation, if no regularity conditions are imposed on them? 80. SOLUTIONS 7.9 - 8.1 The answer is no. The equation (1) in our solution turns out to be equivalent to the system of equations o(z+y)=9(z)+9(y) and —_g(zy) = o(z)a(y). (8) A proof of this fact can be found in the paper by J. G. Dhombres, Relations de dépendance entre les équations fonctionnelles de Cauchy in the journal Aequationes Mathematicae 35 (1988), pp. 186-212 (Th. 5). A function g which satisfies the second equation of (8) has the property that. g(2) = (9(2))’, (9) and therefore it assumes nonnegative values on nonnegative numbers. This, combined with the first equation of (8), implies that g is nondecreasing; and since the first equation leads (in a standard way) to the conclusion that 9(2) = g(1)z on the rationals, the same conclusion is forced on the reals. By (9), a(1) equals 0 or 1. Hence g(2) = 0 or g(z) = 2, hence f(z) must be equal to either 1 or z +1. 8.1 Let a, b, c be pairwise distinct numbers, different from zero and such that a+6+¢=0. Prove that (S52 SY Solution ‘The following identity is easily verified: rr ———=————C—t— + v z wyz Set c= b—c, y=c—a,z=a-—bin (I). In view ofatb+e=0 we have y—z=b+c—2a=~3a, z—2 = —30, r—y = —3c, and (1) becomes =3a bre (~8a)(~38)(—36) ~ b= oNe= a\a=8)' Yabe (b= c)(e— a)(a—b)* aX b_ b= oe b abe Multiplying the last two equalities we get the claimed one. SOLUTIONS 8.2 - 8.3 81 8.2 Suppose that n persons P;, P2,...,P, meet at a party. Assume that P, knows 4 persons, P, knows 5 persons, and so on, Pa—¢ knows n — 3 persons. Further assume that each of Pa_s, Pa—4; Pn3 knows n—2 persons, and each of P,—2, Pa—1, Pa knows n—1 persons. Find all integers n > 8 for which this is possible. (It is understood that “to know” is a symmetric nonreflexive relation: if P; knows P; then P; knows P,; to say that P; knows p persons means: knows p persons other than himself/herself.) Solution Assume n> 10. P; has four acquaintances, Pp has five acquaintances, and Py2, Pai, Pa know everybody. Thus P; knows exactly one person and P) knows exactly two persons other than Px», Pai, Pa. Consider the triple Pa_s, Pa—a, Pas. Each of them misses just one other person. It follows that exactly one person among Pn-s, Pa—4; Pas knows P;, but not P2, while the other two know P2, but not P;. Thus P, and P, have no acquaintances P, with k 10). Hence P3 knows all P,s with k > n — 6 (and there are seven of them). This, however, contradicts the assumption that P; knows exactly six persons at the party. Consequently n = 8 or n = 9. And for these two values of n the conditions of the problem can be realized as follows: Case (n= 8). Ps, Pr, Pa know everybody; Ps, Ps know everybody except Pi; Pa knows everybody except Pz. Then P, knows four people, Pz knows five people, Ps knows six, as needed. Case (n= 9). Pr, Ps, Po know everybody; Ps, Ps know all except Pi; P4 knows all except P,; Ps knows all except P; and P2. This is the required configuration. 8.3 In a convex quadrilateral of area 1, the sum of the lengths of all sides and diagonals is not less than 4 + V8. Prove this. Solution If ABCD is the quadrilateral, write a = AB, b = BC,c = CD, d= DA, p= AC, q = BD; assume that the diagonals intersect under an angle of size y. The (unit) area of the quadrilateral is equal to (pg/2)sing, and hence does not exceed pq/2. Thus pq > 2. ‘Triangles ABC and CDA have areas (ab/2)sin B and (cd/2) sin D, not exceeding ab/2 and cd/2, respectively. So ab d (area of ABCD) < F +5. 82 SOLUTIONS 8.3 ~ 8.4 Similarly, 1 = (area of ABCD) = (area of BCD) + (area of DAB) < x + 2, and hence ab+be+ced+da> 4. Consequently, (p +9)? = 4pq + (p — 9)? > 4pq > 8, (atb+etd)? =4(ate)(b+d) + ((ate)—(b+))? 2 4(a+c)(b +d) = 4(ab + be + cd + da) > 16, and so a+bt+c+dt+p+q2>Vi6+ V8 =4+ VB. 8.4 Find all pairs of real numbers z, y satisfying the system of equations ai +y?—ay— fe=0, yt t2?— 25y- By=0. Solution Multiply the first equation by y and the second by z. Then add and subtract the two equations that result: { a +y¥ — fry =0, ( Qety +P — 2ny! —29=0. Equivalently, writing s = 2 +y,q=2y: 4s(s? — 39) —9q =0, ears @) (= P)Qr-1)=0. Ifc = y then the first equation of (1) becomes 823 = 92, yielding « = 0 or z= 9/8. If z # y then the second equation of (2) implies g = 1/2, which inserted into the first equation of (2) gives 8s? — 12s — 0, ie., (2s — 3)(4s? +65 + 3) = 0, with the unique real root s = 3/2. The system tty=s=%, sy=q=}h has solutions z = 1, y= 1/2 and z= 1/2,y=1. Thus the general solution of the original system is constituted by the four pairs (z, y): (0,0), (8,8), (443), G9). SOLUTIONS 8.4 ~ 8.5 83, Remark There are six further solutions in complex numbers, namely (2,58), (30,8), (8a, 8), where a = }(-1+i¥3), 6 = 4(-1—iV3) or vice versa; for details see T. H. Wang’s solution in the problem solving journal Cruz Mathematicorum 17 (1991), 5 (577 King Edward, Ottawa, Ontario, Canada KIN 6N5). 8.5 We are given a certain number of identical sets of weights; each set consists of four different weights expressed by natural numbers (of weight units). Using these weights we are able to weigh out every integer mass up to 1985 (inclusive). How many ways are there to compose such a set of weight sets given that the joint mass of all weights is the least. possible? Solution Let a, b, c, 1 be the weights, from largest to smallest (clearly, the smallest weight must be 1). Suppose there are k such sets. The given conditions imply: a>b>c>l, (1) e (k +1)a, then p =k, and hence m! = m—pa=m-—ka< 1985—ka 10, the equality in (3) forces k = 5,a+b-+c+1 = 397. Conditions (2) then yield ¢ < 6, b < 36, a < 216, whence a+b+c+1 < 259, a contradiction. ‘Try 1986 next, with prime factorization 2-3-331. The sum a+b+c+1 has to be equal to one of the four divisors of 1986 which are not less than 10, namely 331, 662, 993 and 1986. The three largest numbers lead to contradiction, in a manner similar to the case of 1985. So let us try a-+b-+¢+1 = 331, k = 6; conditions (2) become cS7, b<6c+7, a<6b+6c+7. (4) Assuming ¢ < 5 we obtain b < 37, a < 259 and a+b+c+1 < 302, a contradiction again. This leaves 6 and 7 as the possible values of c. Let c = 6. Then by (4) 5 < 43. The values 43, 42, 41 produce the following triples (a, b, c): (281, 43,6), (282, 42,6), (283, 41, 6), all of them satisfying (1), (2), (3) with k = 6. Any 6 < 40 would not do, for then a < 283 (by (4)) and a+b+e+1< 330. Finally, let c = 7. Then by (4) 6 < 49. Values 6 < 39 are eliminated like in the last case. The other ten values of 6 give rise to the ten triples (a, 6,): (323-6, 6,7), b= 40,41,...,49, all of which are good with k = 6 Thus there are 13 ways to compose the sets of weights with properties as required. 8.6 Let P be a point inside a tetrahedron ABCD and let Sa, Sp, Sc, Sp be the centroids (ie. centers of gravity) of the tetrahe- dra PBCD, PCDA, PDAB, PABC. Show that the volume of tetrahedron S4SpScSp equals 1/64 the volume of ABCD. Solution Denote by Ga, Ga, Gc, Gp the centroids of the respective faces BCD, CDA, DAB, ABC, and by G the centroid of ABCD. The centroid of a tetrahedron lies on each one of the four segments joining the vertices to the centroids of the opposite faces and partitions each of these segments in ratio 3: 1. So we have the vector equalities GGA = -} GA, GB, — = (1) GG = -4-GG, GD. SOLUTIONS 8.6 - 8.7 85. ‘The same argument applied to the tetrahedron PBCD yields the equality 5aGa = -4-5,P, which can be rewritten as PSa = 2-PGa. Considering the four tetrahedra PBCD, PCDA, PDAB, PABC we thus have PR =3-PGA, PSp = 3: PGs, (2) PSc =8-PGe, PSp = 8-PGp. Equalities (1) show that the tetrahedron GaGgGcGp is the homothetic image of ABCD; point G is the center of homothety, the ratio equals —}. Similarly, equalities (2) show that S4SgScSp is the homothetic image of GaGaGcGp, with center P and ratio 3. Consequently, tetrahedron S4SpScSp is similar. to ld) |-3|-|3] = 4. Hence the ratio of volumes equals (1)° = & 8.7 Find an upper bound for the ratio 2122 + 2rorg3 + 2324 ap +03+23 +23 over all quadruples of real numbers (21,22, 23,24) # (0,0, 0,0). Note. The smaller bound, the better the solution. Solution It is not hard to work out the exact upper bound for the quantity under consideration; denote this expression by F(21, 22, 23, 24) Write /23 + 23 =u, 2? +23 =0;s0u>0,v>0,ut+v>0. Then Qeorg =u? — (22-23)? < uv? qQ and, by the inequality of Cauchy-Schwarz (see the solution to problem 6.5), 2122 + 23rq < uv. (2) Hence e wun P(21,20,25,24) < BTM =: Olu). @) For u = 0 we have G(0,v) = 0. For u> 0, 1 wou?» du _ iy Btu uy Gun) = ule + u) vtu uo tote Setting w = (v+u)/u, 2 agg Tt S = BVE4 (Vw - Va)? > 23; (4) and hence, in view of (3), Ss 1 _ 1+ 22-20 0°«2 &) F(z1, 22, 23,24) < Gu, v) < 86 SOLUTIONS 8.7 ~ 8.9 Equality in (5) is possible; it holds if and only if (1), (2) and (4) become equalities. Equality in (4) requires w = V2, ie., v = (V2 —1)u. Inequalities (1) and (2) turn into equalities when x2 = 3 and 2; = 24. ‘Thus the number 4(1+¥2) is the least upper bound of F(21, 22,3, 24); attained e.g. at 22 =23=1, 2) =2,=V2-1. 8.8 A convex n-gon ApA;...4An—1 has been partitioned into n — 2 triangles by certain diagonals not intersecting inside the n-gon. Prove that these triangles can be labeled Ai, Az,..., An—2 in such a way that A; is a vertex of Aj, for i = 1,...,n—2. Find the number of all such labelings. Solution The labeling with the given property is unique. We will prove this by induction. For n = 3 the claim is obvious. Fix n > 4 and assume the claim is true for any m-gon, 3< m is SOLUTIONS 8.9 - 9.1 87 a rectangle or there is no such side. In the first case, let C be any one of the endpoints Ci, C2. In the second case, the (open) strip limited by the two lines passing through By, By and perpendicular to B; By contains some vertices of the polygon; choose any one of them and denote it by C. Let £; be the portion of the boundary of the polygon, connecting C to By, and let Co be the portion of the boundary, connecting C to Bz (so that B, ¢ Lo, Bo ¢ £1). Let Dy € L; and Dz € Lz be the two vertices adjacent to C (it can happen that D, = By or Dy = Bz). On the internal bisector (6) of angle DiCD» choose a point Q with 0 < QC < min{QD,,QD2}, close enough to C in order that the foot F of the perpendicular dropped from Q to line B, Bz lies strictly between B, and Bo (this property of points on 6 near C is ensured by the definition of C). \. D, 4 BF By Now, let A; be the vertex on £; whose distance from Q is a maximum (Ai # C because QC < QD;). Then all vertices on £;, and also the point F, lie inside or on the circle with center Q and radius QA;. Con- sequently, ray AiQ makes acute angles with the two sides of the polygon adjacent to Ay. Similarly, the vertex Az on C2 with maximum distance from @ has the analogous property. We need a third such vertex, As. But this is easy: just take vertex C to be Ag. Since Q lies on the bisector of angle C, the angles QCD; and QCD2z are acute. 9.1 We are given a non-right-angled triangle A, AzA3. Points Oi, Oo, Os are the centers of circles Ii, In, I's, pairwise tangent (inter- nally or externally), circle I; passing through A» and As, circle passing through A3, Ai, circle [3 passing through Ay, Ap. Given that the triangles A; A7A3 and O;0203 are similar, determine their angles. 88 SOLUTION 9.1 Solution Let £A;, £0; (i = 1,2,3) be the angles of the two triangles. Thus we have the set equality {2A,, ZA2, £43} = {201, O02, 403}. Two cases are possible: Case 1. The three circles are pairwise externally tangent. Then each A; lies on the corresponding segment O;—10;41 (numbering modulo 3) and therefore 180° = LO, A3A2 + £A3 + £4, A302 = 4(180° — 201) + 23 + 3(180° — 202), whence £O3 = 180° — 22.43; in general, 2O; =180°-22A;, i= 1,2,3. Assuming (without loss of generality) ZA: < LAz < LAs, we have the inequalities LO, > LO > £Oz, so that LA, = 203, LAr =LO2, LA = LO. This yields the system of equations 180° 224A; = Agi, 7 1,2,3, with the unique solution LA; = £Ag = £A3 = 60° SOLUTIONS 9.1 ~ 9.2 89 Case 2. Some circles are tangent internally. Assume that e.g. I, and T, mutually tangent externally, touch I's internally. Then Oz lies on the segment O3Ay, point O; lies on O3Az, and As lies on 0102, and so 180° = ZO, AgAz + £A3 + LA, A302 = $(201) + LAs + $(202), whence LO3 = 2443 — 180° (thus ZAs > 90° and £03 < ZAs). Now, the size of angle A1 420s can be evaluated in two ways: £AyA203 = (180° — £03) = 180° — ZAs, and £A1A203 = LA, AnAg + LA3A201 = LAz + 3(201). Equating the right sides we obtain £0, = 2(180° — 2A3 ~ ZA2) = 22Ai, and likewise LO) = 244. The obtuse angle ZAg is the largest angle in triangle A1A2A3. Since the roles of A, and Ap are symmetric, we may assume LA; < LAz. Then £0, < LO». Since LO3 < LAs, we infer 402 = LA > 90°. Consequently £02 > ZO, whence ZAz > ZA1. Also, 201 =2LA; > ZAj. It follows that £03 = LA, < LO, = LA2 < £02 = LAs. ‘This together with the former equalities yields the system of equations 224A, =2An, 2LA2= LAs, 24 As — 180° = Z Ai, with the unique solution 2A, =4-180°, LAg = 3- 180°, ZAg = 4 -180°. 9.2 — The monic polynomial P(z)= 2" +an-12"*) +++. +012 +49 of degree n > 1 has n distinct real negative roots. Prove that a1 P(1) > 2n?a9. Solution Denote the roots by —21,...,—@n, so that. P(t)=T]@+2:), 2 >0, iat By Vidte’s formulas, Ils. ag 90. SOLUTIONS 9.2 ~ 9.4 Hence, by the arithmetic mean-geometric mean inequality, n>e(Iv)"=9(f8) "=r" the inequality is strict because the y;s are pairwise distinct. For every positive « we have from the binomial formula (1+ 2)" >1+nz+ 4n(n—1)2? > nz (2) (the last inequality is justified by subtracting 2nz from the expression be- tween the two “greater than or equal to” signs and examining the quadratic trinomial that results). Setting in (2) ¢ = 2; (i= we obtain n) and multiplying out the n inequalities P(l)= Te +2) > T=)" = 2nay!”. (3) ist a1 Now it remains to multiply (1) by (3) to establish the claimed inequality. 9.3 Each point in space is colored either blue or red. Show that there exists a square with edge length 1, having exactly 0, 1 or 4 blue vertices Solution Choose any red point P in the space (if there is no such point, there is nothing to do). Consider the sphere S with center P and radius V2. If S is entirely blue, we can easily find a unit square with 4 blue vertices. Thus suppose S has red points and let @ be one of them. Consider the regular octahedron PABCDQ with edge length 1, points P and Q being its antipodal vertices. The unit square ABCD either has 4 blue vertices (and we are done) or it has at least one red vertex; say, A. Then the unit square APCQ has either 0 or 1 blue vertices. 9.4 — Find all triples of positive integers (z, y, z) such that a Solution Suppose (2, y, z) is such a triple. Write = 2*u, y = 2!v, with u, v odd integers. Assuming k > 1, we have 9200 = (gy)? (aly) f#1 = get) where w = (2*~'y)"*? — y*44 is an odd integer greater than 1 (because z+1 > 2). This is impossible, and hence & < I. In a similar way we show SOLUTIONS 9.4 - 9.5 91 that k>l. Sok =1 and we have z = 2'u, y= 2'v, u>v, HED (y= 0) Sout, 50 9300 = (aby)? _ (pty) #2 The sum occurring in the last expression consists of z + 1 odd summands, hence z+ 1 must be even. Write z + 1 = 2t, to obtain 2100 — (atu) — (atv) = 27 (ut — ot) (ut + of) (1) ‘The last two factors have to be some powers of 2; thus ubf-of=2™, ub +ot'= 2", O2; un equivalently, at eget Now we can rewrite (1) as 2200 = 978 (y — v)(u + v) = 22H, Hence 2k +n = 99; and since n > 2, we get 0 << 48. Thus, finally, 99-2k, u= 29-7 41, 298-2k _ 1, and consequently ea y= 298-F 4 OF, y = hy = 298-# — ob , z=1, kaninteger, 0 r. On the other hand, tetrahedron T is also similar to 7’, in ratio 3. Sphere S’ is circumscribed about 7", hence its radius equals 1/3 the circumradius of T: r= R/3. So we get R/r > 3. It is not difficult to see that every number > 3 can be attained as a value of R/r for some tetrahedron T € M. To show this, consider the rectan- gular parallelepiped whose two parallel bases are congruent unit squares ABCD and A'B'C'D’, distance t apart (t = AA’ = BB’ = CC’ = DD’, a varying real parameter). Let T be the tetrahedron AB’CD’. Its circum- center and incenter coincide with the center of symmetry of parallelepiped ABCDA'B'C'D’; thus T is in class M. The circumradius and inradius are easily calculated: _V2+? t SOLUTIONS 9.6 ~ 9.7 93 ‘The ratio R/r = (1/t),/(2+)(1 +20) is a continuous function of the variable t, attaining value 3 at t = 1 (T' then becomes the regular tetra- hedron) and tending to infinity as + 0 or ¢ —+ 00; hence it attains every value > 3. So the range in question is the interval [3, 00). Remark It follows from the solution that the bound R/r > 3 is valid for every tetrahedron, not necessarily from class M. 9.7 Let k and n be integers with 0 < k < n?/4. Assume that k has no prime divisor greater than n. Prove that n! is divisible by k. Solution Let p be any prime divisor of k; then p < n by hypothesis and so p is also a divisor of n!. Suppose p enters k and n! in powers a and f, respectively (that means, k is divisible by p®, but not by p*t, and n! is divisible by p®, but not by p*t#). It will be enough to show that a < . Let m be defined by the condition p” 4k > 4p*, (2) and hence n > 2p%/? > 2p7. This in view of n < p™+} leads to the inequality prtinr 52, Therefore . if p=2, then y3, then y 2p, which, inserted into formula (1), implies >See] =0 46, 4) i=l Skee"), where 94 SOLUTIONS 9.7 ~ 9.8 Note that (in view of (3)) s={j if p=2, 0 if p>3, while a= ado > 2c at 2 >; fa the last inequality is ee for every integer y > 2. Thus by (4) we have B22; (5) equality could only (perhaps) be possible when y < 1 and p> 3. If y= 0, then (5) is certainly strict (@ > 1 because p 2 unless y=1 and p>3. Now, if a is even (a = 27), then by (5) 8 > a, as needed; if a is odd 2y + 1), but strict inequality holds in (5), we are done, as well. It remains to consider the case where y = 1, p> 3 anda =2y+1=3. From (2) we have n? > 4p, i.e., n > 2p9/?, and hence n/p > 2p'/? > 3. Returning to (1) we obtain # > {n/p] > 3 = a and the proof is complete. Remark Here is a proof of the Legendre formula (1). Obviously, # is the sum of the exponents with which p enters 1,2,3,...,n. Write g; = [np7'] for ,m+1 (thus gm41 = 0). Note that, ‘for each i < m+ 1, the set ,n} contains exactly g; integers divisible by p'. So there are exactly Gi—4i41 integers divisible by p', but not by p'+? (in this set). Consequently, B= (q1 — 92) 1+ (92 — 93) 2+ +++ + (Gm — dmti)-m Sn tate + amy as claimed. 9.8 Pairwise distinct real numbers are arranged into a rectangular array with m rows and n columns. In each row the entries are arranged increasingly (from left to right). Each column is then rearranged in decreasing order (from top to bottom). Prove that in the reor- ganized array, all rows are again arranged increasingly. Solution Suppose, to the contrary, that there exist indices j € {1,...,m} and kl {1,...,n}, & < 1, such that the jth row of the new array forms a sequence (21,...,2n) with zz > a). Let () ~ () SOLUTIONS 9.8 - 9.9 95. be the & th column and the /th column of the original array; by assumption, a > Bp(m) 5 Bacay > +++ > Byte) - Thus, if ie {1,. if ie sd}, then apc: > apy) = te > 215 ym}, then ayy < bai) < by(j) = 1 It follows that the m-element set {a1,...,@m} contains at least j numbers greater than z; and at least m—j +1 numbers smaller than 2). This is obviously a contradiction. 9.9 Determine all continuous monotonic functions f : R —+ R. which satisfy f(1)=1and f(f(c)) = (f(z) for all 2 eR. Solution Denote by J the set of all values taken by f. Since f is continuous and monotonic, J is an interval (finite or not). The equation imposed on f can be rewritten as f(z)=2? for zed. Clearly, 1 = f(1) € J. Suppose that J contains a negative number u. Then [u, 1] is a subinterval of J; a contradiction because the squaring function is not monotone on such an interval. Thus J C [0,0o), which means that f(c)20 forall 2eR. Choose any a € J. Then a? = f(a) € J, a4 = f(a?) € J, and so on, a” € J for n = 1,2,3,.... Ifa < 1, this sequence tends to zero (and thus 0 J, by the continuity of f); if a> 1, this sequence tends to infinity. In other words: if J contains numbers smaller than 1, then 0 is the left endpoint of J, and if J contains numbers greater than 1, then oo is the right endpoint of J. Thus J is one of the following four sets: {1}, (0, 1), [1, 09), [0, 00). 96 SOLUTIONS 9.9 - 10.1 Accordingly, f is given by one of the following four formulas: f(@e)=1 forall ceR, 0, 2<0, see)= $2?) 0Se<1, aie ata 1 a 4. Write p instead of 1987. (In general, p may be any prime such that ged(n, p— 1) = 1; p = 1987 has this property, as 1986 = 2-3-331, while all prime divisors of n have an even number of digits by assumption.) Suppose, contrary to the assertion, that the equality P(x) = P(y) holds for some rationals z = a/c, y = b/d, # y; assume that a,b,c,d are integers with c,d > 0, ged(a,c) = ged(b,d) = 1. Equality P(x) = P(y) says that d°(a" — pac?) = c®(6" — pbd”~). Q) The expressions in the parentheses on the left side and on the right side of (1) are coprime to ¢ and d, respectively; so c = d, and hence a ¢ 8. Equality (1) becomes b" — a" = p(b— ajc"). (2) Division by 6 — a yields (3) It follows from (2) and (3) (a being coprime to c) that a and 6 are nondivisible by p. Thus be=1 (mod p) (4) holds for some e € {1,2,...,p— 1}. Hence by (2) (ae)" = a"e" = bre" = (mod p). (5) Two cases are now possible. Case (ae #1 (mod p)). By (5), ae # 0 (mod p), and hence, by virtue of Fermat’s theorem (see the Remark, below) (aeP"!=1 (mod p). (6) 98 SOLUTION 10.2 Let @ be the least positive exponent for which (ae) = 1 (mod p); clearly, @ > 1, and it follows from (5) and (6) that a is a common divisor of n and p—1; acontradiction. Case (ae = 1 (mod p)). Then by (4) a=b#0 (mod p). (n Thus equality (3) yields na”~! = 0 (mod p), whence n = 0 (mod p). Since nis a perfect square, n=0 (mod p?) (8) Write, in agreement with (7), a = kp+r, 6 = Ip+r. By the binomial formula, a’ =ikpr’}4r! (mod p*), = jlpri-t +7 (mod p?), and hence abl So the sum on the left side of (3) is congruent modulo p? to n(n—1) 2 ik + j)prit3-) 499 (mod p?). act Yo (ikt (n= 1- lpr? turd = (k + prt? + nr? io In view of (8), this number is divisible by p; so the product on the right of (3) must be divisible by p?, which means that p divides c. Thus (2) becomes oa" =(b—a)p"w, —_w an integer. (9) Assume that p enters the difference b— a in a power m > 1 (see (7)): b-a=p™q, m2>1, q#0 (mod p). (10) Now, let s be a positive integer not divisible by p. We claim that for every integer z > 0 there exists an integer t, with ote? — gt?’ = p™t4,, — t, #0 (mod p). (11) We induct on z. For z = 0 we have by (10) and the binomial formula bf —a® = (a+ qp™)* —a* = sa‘~!qp™ + Ap’, Aan integer; so we can set to = sa*'qg-+ Ap™ #0 (mod p). Fix z > 0; assume ¢, exists and (11) holds. Consider z + 1: perth _ gspttt _ (ory? _ (a?*)? = (a°** +0™4,)? — (ay? P sy () aiF (Pi) lms) gh Fe SOLUTIONS 10.2 ~ 10.3 99 In this sum we isolate the first term and the last term; all the other sum- mands are divisible by p!+2("+#) (because the binomial coefficients are divisible by p). So we have fe pat? (P-Lpmtz4, 4 Bpltomt2 4 port? ae , B an integer, and it suffices to set tog = at ODE, + Bp™* 4 l™*NP-“D-1 40 (mod p) to obtain representation (11) for z + 1 in place of z. This completes the proof of claim (11). Write the given integer n in the form = ope eee is 0, ceeesodl isie10.\(modap): (12) Apply equality (11) to these specific values of s and z (and the suitable t,). Equating the right sides of (9) and (11), and using (10), we obtain pt, =p" qu. Since t, # 0 (mod p), this implies n < z, while (12) forces n > p* > z. This is the desired contradiction, showing that the equality P(z) = P(y) cannot hold for any distinct rational numbers « and y. Remark Fermat’s “Little” Theorem says that if p is a prime and w is an integer nondivisible by p then u?-? = 1 (mod p). Here is an outline of a proof: it is enough to consider u € {1,...,p — 1}. Induct on u, expanding (u + 1)? binomially and using the fact that the binomial coefficients @), O0 0. Suppose there exists a positive integer m such that the difference tm — 20 equals k, also an integer. Show that the limit lim(z,/n) exists; express its value in terms of the given data. Solution ‘The functional equation satisfied by f implies f(et+)=f(z)+l for ceR, lez (1) (use induction on I = 0,1,2,...; the case of negative | follows easily from that for positive !). A next quick induction proof shows that tngm=@ntk for n=0,1,2,... (2) (induct on n starting from tm = 20 +k and using (1)). 100 SOLUTIONS 10.3 ~ 10.4 The third claim is eee ny | toca Oy, (3) Considering r to be fixed, induct on q. For q = 0, (3) holds. Assume (3) for some q > 0 and pass tog +1: Lrpm(g41) = Lrtmgem = Cr¢mg th = ar thgtk=2p+k(q+1); we have used (2) with n = r+ mg, and the inductive assumption. Thus, (3) follows. From (3) we are able to derive the solution of the problem. Take any positive integer n; divide it by m, denote (the integer part of) the quotient by q(n) and the remainder by r(n). Then by (3) Zn _ _Tr(n)tmaln) _ Er(ny than) _ k 1+ an m n r(n)-+mg(n) r(n)+mg(n) m 1+f,’ where (n) = Een) = 2th w= Fain)? = agin As n goes to infinity, q(n) does so too, while the numerators take only a finite number of values. Hence a + 0, Bn — 0, and so 10.4 Does the set {1,2,...,3000} contain a subset A consisting of 2000 numbers such that 2 € A implies 22 ¢ A? Solution Call a set A with this property double-free. Consider the problem with 3000 replaced by arbitrary N. For every positive odd integer q consider the sequence % 24, 49, 84, .-- qa Out of each pair of consecutive terms of such a sequence, only one can belong to a double-free set. Thus in order to construct a double-free set Amax C {1,2,...,N} with a maximum number of elements, we can assign to Amax every second entry of each sequence (1), starting, for instance, from the least one. So we may define Amax = {n= 4g: n0 integers, g odd} (note that, in general, the set Amax thus defined is not the unique double- free set in {1,..., NV} of maximum cardinality). SOLUTIONS 10.4 - 10.5 101 For a fixed k, the set {1,...,.N} contains [(4-*N + 1)/2] numbers of the form 4*q, with q odd. Thus, if m is such that 4" < N < 4™+1, then the number of elements of Amax equals Solent + yy. @ es For N = 3000, this sum evaluates to 1999. So there is no 2000-element double-free subset of {1,2,...,3000}. Remark If N= oad (4 € {0,1}, =o of N, then the maximum cardinality of a double-free subset of {1,..., N} equals sy) is the binary representation f= 24 Ey ei(-1y (3) = To see that the formulas (2) and (3) indeed define the same number, denote the expression in (2) by g(N), assume (as before) 4" < N <4™+1, m>1, and write N=K-.4"+M with K € {1,2,3}, 0 dy, S contains pairs of points at mutual distance dj; in every such pair, at least one point does not belong to Ay. So we may find a point PE SM Aa Now consider the sphere $’ of radius dz, centered at P. According to condition (1), S’ C A, U Ag. Hence SAS'CAL (2) The intersection SS’ is a circle; denote its radius by r. For every point @ of this circle, OPQ is an isosceles triangle with OP = OQ = ds, PQ = do < dg. Hence 60° < OPQ < 90°, and so r= PQ-sinLOPQ > (V3/2)d2 > do/2, showing that the diameter of circle S'S’ exceeds dp. Thus we can find a pair of points of S175! at distance d;. And this is a contradiction to (1) and (2) 10.6 Let C be the circle with radius 1 and let n > 1 be a fixed integer. For any set A of n points Py,...,P, on C' define D(A) = max(min6(P,,d)) where 6(P,1) denotes the distance from point P to line | and the maximum is taken over all diameters d of circle C. Let F, be the family of all n-element subsets AC C and let Dn = min D(A). AOR, Calculate D, and describe all sets A € F, with D(A) = Dn. Solution For a point P € C denote by P’ the point antipodal to P; for a set ACC denote by A’ the antipodal image of A (ie. A’ = {P's P € A}). Se’ \7 7 Take aset A= {Pi,..., Pn} € Fn. The set AUA’ consists of 2m points, m sin(x/2m) > sin(x/2n). (2) The first inequality in (2) becomes an equality if and only if all ares in A are equal; i.e., when AU A’ is the set of vertices of a regular 2m-gon. The second inequality in (2) is an equality for m =n, ie., when A and A’ are disjoint. The conclusion follows: Dy = juin D(A) = sin(x/2n). The minimum is attained for every set A consisting of some n vertices of a regular 2n-gon inscribed in C, without a pair of antipodal vertices. 104 SOLUTION 10.7 E 10.7 For any natural number n = J a;10° (where a; € {0,1,...,9}, i=0 a, # 0) write i Var-s10 i=0 & Sa, 2 iso Consider the set P={n: n=n*, p(n)/3=s(n)-1} and let Q be the set of all numbers in P with all digits greater than 1. (a) Show that P is infinite. (b) Show that @ is finite. (c) Write down alll the elements of Q. Solution When n= n*, we shall call na palindrome. (a) For any natural & > 2 let k’ = (3-1 — 6k + 1)/2 (an integer). ‘Write a string of k’ ones followed by 2k threes and k’ ones, and denote the resulting palindrome by n. Then s(n) = 6k + 2k’ = 3-141, p(n) = 37, son € P. Since k might be an arbitrary integer > 2, we obtain an infinite sequence of elements of P. (b) Suppose n € Q is an m-digit number. The product p(n) is divisible by 3; therefore at least one digit is > 3. Hence p(n) > 3-277, s(n) < 9m. The equation defining the set P now implies the inequality 2"-! < 9m—1, and so m < 6. (c) According to the conclusion of (b), every element of Q has one of the following forms: (zyzzyz)io, (zyzyz)io, (zyyz)o, (zur), (z)0, (2), with 2,y,z € {2,3,...,9} Case 1. n=(zxyzzyz)19. The equation of P takes the form 2y?2” = 6(z@ +y+2)—3. () Then «, y, z are odd integers; hence they exceed 2; the product on the left side of (1) is > 3° = 729, the right side of (1) is < 6(9 +9 +9) —3 = 159; no solutions.

S-ar putea să vă placă și